You are on page 1of 93

1.

LEE HONG KOK VS. DAVID G.R. No. L-30389, Dec. 27, 1972

FACTS: This is regarding a piece of land which Aniano David acquired lawful title thereto, pursuant to his miscellaneous sales application. After approval of his application, the Director of Lands issued an order of award and issuance of sales patent, covering said lot by virtue of which the Undersecretary of Agriculture and Natural Resources issued a Miscellaneous Sales Patent. The Register of Deeds then issued an original certificate of title to David. During all this time, Lee Hong Kok did not oppose nor file any adverse claim. ISSUE: Whether or not Lee Hong Kok may question the government grant. HELD: Only the Government, represented by the Director of Lands or the Secretary of Agriculture and Natural Resources, can bring an action to cancel a void certificate of title issued pursuant to a void patent. This was not done by said officers but by private parties like the plaintiffs, who cannot claim that the patent and title issued for the land involved are void since they are not the registered owners thereof nor had they been declared as owners in the cadastral proceedings after claiming it as their private property. The fact that the grant was made by the government is undisputed. Whether the grant was in conformity with the law or not is a question which the government may raise, but until it is raised by the government and set aside, the defendant cannot question it. The legality of the grant is a question between the grantee and the government.

2.

CARINO VS. INSULAR GOVERNMENT 41 PHIL 935

FACTS: An Igorot applied for the registration of a certain land. He and his ancestors had held the land as owners for more than 50 years, which he inherited under Igorot customs. There was no document of title issued for the land when he applied for registration. The government contends that the land in question belonged to the state. Under the Spanish Law, all lands belonged to the Spanish Crown except those with permit private titles. Moreover, there is no prescription against the Crown. ISSUE: Whether or not the land in question belonged to the Spanish Crown under the Regalian Doctrine. HELD: No. Law and justice require that the applicant should be granted title to his land. The United States Supreme Court, through Justice Holmes declared: It might perhaps, be proper and sufficient to say that when, as far as testimony or memory goes, the land has been held by individuals under a claim of private ownership, it will be presumed to have been held in the same way from before the Spanish conquest, and never to have been public land. There is an existence of native title to land, or ownership of land by Filipinos by virtue of possession under a claim of ownership since time immemorial and independent of any grant from the Spanish Crown, as an exception to the theory of jura regalia.

3.

CRUZ VS. SEC OF DENR 347 SCRA 128 (2000)

FACTS: Cruz, a noted constitutionalist, assailed the validity of the RA 8371 or the Indigenous Peoples Rights Act on the ground that the law amount to an unlawful deprivation of the States ownership over lands of the public domain as well as minerals and other natural resources therein, in violation of the regalian doctrine embodied in Section 2, Article XII of the Constitution. The IPRA law basically enumerates the rights of the indigenous peoples over ancestral domains which may include natural resources. Cruz et al contend that, by providing for an all-encompassing definition of ancestral domains and ancestral lands which might even include private lands found within said areas, Sections 3(a) and 3(b) of said law violate the rights of private landowners. ISSUE: Whether or not the IPRA law is unconstitutional. HELD: The SC deliberated upon the matter. After deliberation they voted and reached a 7-7 vote. They deliberated again and the same result transpired. Since there was no majority vote, Cruzs petition was dismissed and the IPRA law was sustained. Hence, ancestral domains may include public domain somehow against the regalian doctrine.

4.

STA. ROSA MINING VS. LEIDO 156 SCRA 1 (1987)

FACTS: Presidential Decree No.1214 was issued requiring holders of subsisting and valid patentable mining claims located under the provisions of the Philippine Bill of 1902 to file a mining lease of application within one (1) year from the approval of the Decree. To protect its rights, petitioner Santa Rosa Mining Company files a special civil action for certiorari and prohibition confronting the said Decree as unconstitutional in that it amounts to a deprivation of property without due process of law. Subsequently, three (3) days after, petitioner filed a mining lease app lication, but under protest, with a reservation that it is not waiving its rights over its mining claims until the validity of the Decree shall have been passed upon by the Court. The respondents allege that petitioner has no standing to file the instant petition and question the Decree as it failed to fully exhaust administrative remedies.

ISSUE: Whether or not Presidential Decree No. 1214 is constitutional.

HELD: Yes, Presidential Decree No. 1214 is constitutional, even assuming arguendo that petitioners was not bound to exhaust administrative remedies for its mining claims to be valid in the outset. It is a valid exercise of the sovereign power of the State, as owner, over the lands of the public domain, of which petitioners mining claims still for m a part. Moreover, Presidential Decree No. 1214 is in accord with Sec. 8, Art XIV of the 1937 Constitution.

5.

SAN MIGUEL CORPORATION VS. COURT OF APPEALS (GR # 57667, May 28, 1990)

FACTS: This is a petition for review on certiorari where petitioner San Miguel Corporation who purchased Lot 684 from Silverio Perez, seeks the reversal of the decision of the Court of Appeals denying its application for registration of the said land in view of its failure to show entitlement thereto. The Solicitor General opposed and appealed the application contending that the land in question is part of public domain and that petitioner being a private corporation is disqualified from holding alienable lands of the public domain. In this case, petitioner claims that its predecessor-in-interest had open, exclusive and undisputed possession of the land in question based on documentary evidence of tax declarations and receipts, and testimonial evidence of vendor Silverio Perez. ISSUE: Whether or not the evidence presented by the petitioner is sufficient to warrant a ruling that petitioner and/or its predecessor-in-interest has a registrable right over Lot 684. HELD: No. Documentary evidence of tax declarations and receipts are not conclusive evidence of ownership or right of possession over a piece of land but mere indicia of a claim of ownership. They only become strong evidence of ownership of land acquired by prescription when accompanied by proof of actual possession. Also, the testimony of vendor Silverio Perez as proof of actual possession is weak and was not corroborated by other witnesses.

6.

ALMEDA VS. COURT OF APPEALS GR No. 85322, April 30, 1991

FACTS: Petitioner Jose Almeda filed a notice of appeal which was disapproved by the trial court due to it being filed five (5) days late beyond the reglementary period and subsequently denied of motion for reconsideration. Respondent court dismissed the petition contending that the requirement regarding perfection of an appeal was not only mandatory but jurisdictional such that the petitioners failure to comply therewith had the effect of rendering the judgment final. Subsequently, petitioner motions for reconsideration and is denied. Also, it was found that there was lack of merit in the petitioners reason for the late filing of the notice of appeal.

ISSUE: Whether or not failure to comply with the requirement regarding perfection of an appeal within reglementary period would render a judgment final and executory.

HELD: Yes. The period to appeal is prescribed not only by the Rules of Court but also by statute, particularly Sec 39 of BP 129, which provides: Sec.39. Appeals. The period for appeal from final orders, resolutions, awards, judgments, or decisions of any court in all cases shall be fifteen (15) days counted from the notice of the final order, resolution, award, judgment, or decision appealed from The right to appeal is a statutory right and one who seeks to avail of it must strictly comply with the statutes or rules as they are considered indispensable interdictions against needless delays and for an orderly discharge of judicial business. Due to petitioners negligence of failing to perfect his appeal, there is no recour se but to deny the petition thus making the judgment of the trial court final and executory.

7.

DIRECTOR OF LANDS VS. KALAHI INVESTMENTS, INC., GR No. 48066, January 31, 1989

FACTS: This is an appeal from the decision of the Court of First Instance (now Regional Trial Court) of Pampanga, denying the application of Kalahi Investments, Inc. (Kalahi, for short) for registration of Lot No. 1851-B of the Florida blanca Cadastre. In its decision, the Court of Appeals found the following facts to be established by the evidence: On December 12, 1963, Kalahi Investment, Inc. moved for an advanced hearing of Lot No. 1851-B, Florida blanca Cadastre. Evidence was presented and Kalahi's title was to be registered under the provisions of Act 496. The Bureau of Forestry's opposition is based on the ground that these lands are part of the vast public forest, known as TIMBER LAND of Project No. 11, Exhibit 4, and Director of Forestry. Until now these lands are not released by the proper authorities as alienable agricultural lands; instead on August 9, 1966, the President of the Philippines issued Proclamation No. 82, declaring these lands as part of the Mt. Dorst Forest Reserve.

ISSUE: Whether or not mining claims, acquired, registered, perfected acted, and patentable under the Old Mining Law, mature to private ownership which would entitle the claimant-applicant to the ownership thereof.

HELD: Mere location does not mean absolute ownership over the affected land or the located claim. It merely segregates the located land or area from the public domain by barring other would be locators from locating the same and appropriating for them the minerals found therein. To rule otherwise would imply that location is all that is needed to acquire and maintain rights over a located mining claim. This, the Court cannot approve or sanction because it is contrary to the intention of the lawmaker that the locator should faithfully and consistently comply with the requirements for annual work and improvements in the located mining claims. It is not clear if claimant Kalahi has fully complied with the requirements of the Act of Congress of 1902. This is a factual issue which is not within the scope of our jurisdiction. The records show that claimant has already filed a mining lease application (p. 357, Record on Appeal). Its mining claims, therefore, are deemed covered by P.D. 1214, and the Bureau of Mines may,

accordingly process the same as a lease application, in accordance with P.D. 463, pursuant to Sec. 2of P.D. No. 1214. It is understood of course that prior to the approval of the lease application, the applicant must show that it has fully and faithfully complied with the requirements of the Philippine Bill of 1902, in effect upholding the dissenting opinion of Justice Concepcion in the Gold Creek Mining case. As to whether or not the Bureau of Mines is likewise qualified to rule on whether there has been full and faithful compliance with the requirements of the Philippine Bill of 1902 as amended, the Court ruled that the Bureau of Mines is so empowered as a corollary function in the processing of mining lease applications. The decision of the CFI of Pampanga, (now Regional Trial Court) is hereby AFFIRMED, with the MODIFICATION in that Kalahi's mining claims may be processed as a mining lease application by the Bureau of Mines.

8.

REPUBLIC VS. ENCISO GR No. 160145, November 11, 2005

FACTS: The respondent, alleging to be the owner in fee simple of a parcel of residential land located in Zambales, filed a petition for land registration before the RTC of Iba, Zambales. He acquired title to the said lot by virtue of an extrajudicial settlement of estate and quitclaim on March 15, 1999; the said property is not tenanted or occupied by any person other than the respondent and his family who are in actual physical possession of the same for 30 years. Petitioner Republic of the Philippines, through the Office of the Solicitor General (OSG), opposed the application on the following grounds: (a) neither the respondent nor his predecessors-in-interest have been in open, continuous, exclusive, and notorious possession and occupation of the subject land since June 12, 1945 or prior thereto; (b) the respondent failed to adduce any muniment of title and/or the tax declaration (c) the alleged tax declaration adverted to in the application does not appear to be genuine and the tax declarations indicate such possession to be of recent vintage; (d) the claim of ownership in fee simple on the basis of Spanish title or grant can no longer be availed of by the respondent considering that he failed to file an appropriate application for registration within the period of six months from February 16, 1976 as required by P.D. No. 892; and (e) the subject land is a portion of the public domain belonging to the Republic of the Philippines which is not subject to private appropriation.[5]

ISSUE: For reclaimed land to be registered as private property what is required? HELD: (1) There must be a proof that the land had been classified as alienable; (2) The person seeking registration must showproof of having acquired the property (e.g., by prescription). Well-entrenched is the rule that the burden of proof in land registration cases rests on the applicant who must show clear, positive and convincing evidence that his alleged possession and occupation were of the nature and duration required by law.

Bare allegations, without more, do not amount to preponderant evidence that would shift the burden to the oppositor.[28] Evidently, the respondent failed to prove that the lot was classified as part of the disposable and alienable land of the public domain; and (2) he and his predecessors-ininterest have been in open, continuous, exclusive, and notorious possession and occupation thereof in the concept of owners since time immemorial, or from June 12, 1945.

9.

CHAVEZ VS. PEA GR No. 133250, July 9, 2002

FACTS: In 1973, the Comissioner on Public Highways entered into a contract to reclaim areas of Manila Bay with the Construction and Development Corportion of the Philippines (CDCP). PEA (Public Estates Authority) was created by President Marcos under P.D. 1084, tasked with developing and leasing reclaimed lands. These lands were transferred to the care of PEA under P.D. 1085 as part of the Manila Cavite Road and Reclamation Project (MCRRP). CDCP and PEA entered into an agreement that all future projects under the MCRRP would be funded and owned by PEA. By 1988, President Aquino issued Special Patent No. 3517 transferring lands to PEA. It was followed by the transfer of three Titles (7309, 7311 and 7312) by the Register of Deeds of Paranaque to PEA covering the three reclaimed islands known as the FREEDOM ISLANDS. Subsquently, PEA entered into a joint venture agreement (JVA) with AMARI, a Thai-Philippine corporation to develop the Freedom Islands. Along with another 250 hectares, PEA and AMARI entered the JVA which would later transfer said lands to AMARI. This caused a stir especially when Sen. Maceda assailed the agreement, claiming that such lands were part of public domain (famously known as the mother of all scams). Peitioner Frank J. Chavez filed case as a taxpayer praying for mandamus, a writ of preliminary injunction and a TRO against the sale of reclaimed lands by PEA to AMARI and from implementing the JVA. Following these events, under President

Estradas admin, PEA and AMARI entered into an Amended JVA and Mr. Chaves claim that the contract is null and void.

ISSUE:

Whether or not the transfer to AMARI lands reclaimed or to be reclaimed as part of the stipulations in the (Amended) JVA between AMARI and PEA violate Sec. 3 Art. XII of the 1987 Constitution HELD: On the issue of Amended JVA as violating the constitution: The 157.84 hectares of reclaimed lands comprising the Freedom Islands, now covered by certificates of title in the name of PEA, are alienable lands of the public domain. PEA may lease these lands to private corporations but may not sell or transfer ownership of these lands to private corporations. PEA may only sell these lands to Philippine citizens, subject to the ownership limitations in the 1987 Constitution and existing laws.

10.

LAUREL VS. GARCIA 187 SCRA 797 (1990)

FACTS: The subject Roppongi property is one of the properties acquired by the Philippines from Japan pursuant to a Reparations Agreement. The property is where the Philippine Embassy was once located, before it transferred to the Nampeidai property. It was decided that the properties would be available to sale or disposition. One of the first properties opened up for public auction was the Roppongi property, despite numerous oppositions from different sectors.

ISSUE: Can the Roppongi property and others of its kind be alienated by the Philippine Government? HELD:

The Roppongi property was acquired together with the other properties through reparation agreements. They were assigned to the government sector and that the Roppongi property was specifically designated under the agreement to house the Philippine embassy. It is of public dominion unless it is convincingly shown that the property has become patrimonial. The respondents have failed to do so. As property of public dominion, the Roppongi lot is outside the commerce of man. It cannot be alienated. Its ownership is a special collective ownership for general use and payment, in application to the satisfaction of collective needs, and resides in the social group. The purpose is not to serve the State as the juridical person but the citizens; it is intended for the common and public welfare and cannot be the object of appropriation. The fact that the Roppongi site has not been used for a long time for actual Embassy service doesnt automatically convert it to patrimonial property. Any such conversion happens only if the property is withdrawn from public use. A property continues to be part of the public domain, not available for private appropriation or

ownership until there is a formal declaration on the part of the government to withdraw it from being such.

11.

MINERS ASSOCIATION VS. FACTORAN 240 SCRA 100 (1995)

FACTS: The petition seeks a ruling from this court on the validity of two Administrative Orders 57 and 82 issued by the Secretary of the Department of Environment and Natural Resources to carry out the provisions of Executive Orders 279 and 211. This petition arose from the fact that the 1987 Constitution provided for a different system of exploration, development and utilization of the countrys natural resources. Unlike the 1935 and 1973 Constitutions that allow the utilization of inalienable lands of public domain through license, concession or lease, the 1987 Constitution provides for the full control and supervision by the state of the exploration, development and utilization of the countrys natural resources. Pres. Cory Aquino promulgated EO 211, which prescribes the interim procedures in the processing and approval of applications for the exploration, development and utilization of minerals in accordance to the 1987 Constitution. In addition, Pres. Aquino also promulgated EO 279 authorizing the DENR Secretary to negotiate and conclude joint venture, co-production or production-sharing agreements for the exploration, development and utilization of mineral resources and prescribing the guidelines for such agreements and those agreements involving technical or financial assistance by foreign-owned corporations for large-scale exploration, development, and utilization of minerals. In line with EO 279, the DENR Secretary issued AO 57 Guidelines of Mineral Production Sharing Agreement under EO 279 and AO 82 Procedural Guidelines on the Award of Mineral Production Sharing Agreement (MPSA) through negotiation. Petitioner, Miners Association of the Philippines, mainly contend that the DENR Secretary issued both AOs 57 and 82 in excess of his rule-making power because these are inconsistent with the provisions of EO 279. ISSUE: Whether AO Nos. 57 and 82, which are promulgated by the DENR, are valid and constitutional HELD: AO Nos. 57 and 82 are both constitutional and valid. This is due to the fact that EO 279, in effect, gave the Secretary of Natural Resources the authority to conclude joint venture, co-production, or production sharing agreements for the exploration, development and utilization of mineral resources. Furthermore, the constitutionality of these administrative orders goes to show that the utilization of inalienable lands of

public domain is not merely done through license, concession or lease since the options are now also open to the State through direct undertaking or by entering into coproduction, joint venture, or production sharing agreements.

12.

REPUBLIC VS. ROSEMOOR GR No. 149927, March 30, 2004

FACTS: Petitioner Rosemoor Mining and Development Corporation after having been granted permission to prospect for marble deposits in the mountains of Biak-na-Bato, San Miguel, Bulacan, succeeded in discovering marble deposits of high quality and in commercial quantities in Mount Mabio which forms part of the Biak-na-Bato mountain range. The petitioner then applied with the Bureau of Mines, now Mines and Geosciences Bureau, for the issuance of the corresponding license to exploit said marble deposits. License No. 33 was issued by the Bureau of Mines in favor of the herein petitioners. Shortly thereafter, Respondent Ernesto Maceda cancelled the petitioners license stating that their license had illegally been issued, because it violated Section 69 of PD 463; and that there was no more public interest served by the continued existence or renewal of the license. The latter reason was confirmed by the language of Proclamation No. 84. According to this law, public interest would be served by reverting the parcel of land that was excluded by Proclamation No. 2204 to the former status of that land as part of the Biak-na-Bato national park.

ISSUE: Whether or not Presidential Proclamation No. 84 is valid.

HELD: Yes. We cannot sustain the argument that Proclamation No. 84 is a bill of attainder; that is, a legislative act which inflicts punishment without judicial trial. Its declaration that QLP No. 33 is a patent nullity is certainly not a declaration of guilt. Neither is the cancellation of the license a punishment within the purview of the constitutional proscription against bills of attainder. Too, there is no merit in the argument that the proclamation is an ex post facto law. It is settled that an ex post facto law is limited in its scope only to matters criminal in nature. Proclamation 84, which merely restored the area excluded from the Biak-na-Bato national park by canceling respondents license, is clearly not penal in character.

Also at the time President Aquino issued Proclamation No. 84 on March 9, 1987, she was still validly exercising legislative powers under the Provisional Constitution of 1986. Section 1 of Article II of Proclamation No. 3, which promulgated the Provisional Constitution, granted her legislative power until a legislature is elected and convened under a new Constitution. The grant of such power is also explicitly recognized and provided for in Section 6 of Article XVII of the 1987 Constitution.

13.

LA BUGAL-B'LAAN TRIBAL ASSN. VS. DENR GR127872, Jan 27, 2004, MR GR 127882, Dec. 1, 2004

FACTS: On January 27, 2004, the Court en banc promulgated its Decision granting the Petition and declaring the unconstitutionality of certain provisions of RA 7942, DAO 9640, as well as of the entire FTAA executed between the government and WMCP, mainly on the finding that FTAAs are service contracts prohibited by the 1987 Constitution. The Decision struck down the subject FTAA for being similar to service contracts, which, though permitted under the 1973 Constitution, were subsequently denounced for being antithetical to the principle of sovereignty over our natural resources, because they allowed foreign control over the exploitation of our natural resources, to the prejudice of the Filipino nation. ISSUE: Are foreign-owned corporations in the large-scale exploration, development, and utilization of petroleum, minerals and mineral oils limited to technical or financial assistance only? HELD: Only technical assistance or financial assistance agreements may be entered into, and only for large-scale activities. Full control is not anathematic to day-to-day management by the contractor, provided that the State retains the power to direct overall strategy; and to set aside, reverse or modify plans and actions of the contractor. The idea of full control is similar to that which is exercised by the board of directors of a private corporation: the performance of managerial, operational, financial, marketing and other functions may be delegated to subordinate officers or given to contractual entities, but the board retains full residual control of the business.

14. FACTS:

PHILIPPINE GEOTHERMAL VS. NAPOCOR GR No. 144302, May 27, 2004

The National Corporation entered into a service contract with petitioner ,a corporation organized and existing under the laws of California,USA for the exploration and exploitation of geothermal resources covering the Tivi and Mak-ban Geothermal Fields. Albeit the service contract was to expire in 1996, the negotiations for its renewal started as early as 1994. NPS however doubtful whether a renewal would be constitutional in light of section , Article XII of the Constitution.

ISSUE:

Whether or not the renewal would be constitutional in light of Section 2,Article XII of the Constitution.

HELD:

Since only the issue of jurisdiction over the constitutionality of a contract was elevated to this court it is beyond its jurisdiction to pass upon and approved the compromise agreement of the parties who have as stated agreed to terminate the service contract subject of the dispute in favor of series of agreement that start with provisional interim then transition and finally Geothermal Resources Sale contract the forging of which agreements is intended to effectively erase any doubt as to the legality of the compromise.

15.

JG SUMMIT VS. CA GR No. 124293, January 31, 2005

FACTS: The National Investment and Development Corporation (NIDC), a government corporation, entered into a Joint Venture Agreement (JVA) with Kawasaki Heavy Industries, Ltd. of Kobe, Japan (KAWASAKI) for the construction, operation and management of the Subic National Shipyard Inc., (SNS) which subsequently became the Philippine Shipyard and Engineering Corporation (PHILSECO). Under the JVA, the NDC and KAWASAKI will contribute P330M for the capitalization of PHILSECO in the proportion of 60%-40% respectively. One of its salient features is the grant to the parties of the right of first refusal should either of them decide to sell, assign or transfer its interest in the joint venture. NIDC transferred all its rights, title and interest in PHILSECO to the Philippine National Bank (PNB). Such interests were subsequently transferred to the National Government pursuant to an Administrative Order. When the former President Aquino issued Proclamation No. 50 establishing the Committee on Privatization (COP) and the Asset Privatization Trust (APT) to take title to, and possession of, conserve, manage and dispose of non-performing assets of the National Government, a trust agreement was entered into between the National Government and the APT wherein the latter was named the trustee of the National Governments share in PHILSECO. In the interest of the national economy and the government, the COP and the APT deemed it best to sell the National Governments share in PHILSECO to private entities. After a series of negotiations between the APT and KAWASAKI , they agreed that the latters right of first refusal under the JVA be exchanged for the right to top by 5%, the highest bid for the said shares. They further agreed that KAWASAKI would be entitled to name a company in which it was a stockholder, which could exercise the right

to top. KAWASAKI then informed APT that Philyards Holdings, Inc. (PHI) would exercise its right to top. At the public bidding, petitioner J.G. Summit Holdings Inc. submitted a bid of Two Billion and Thirty Million Pesos (Php2,030,000,000.00) with an acknowledgement of KAWASAKI/PHILYARDS right to top. As petitioner was declared the highest bidder, the COP approved the sale subject to the right of Kawasaki Heavy Industries, Inc. / PHILYARDS Holdings Inc. to top JGs bid by 5% as specified in the bidding rules. On the other hand, the respondent by virtue of right to top by 5%, the highest bid for the said shares timely exercised the same. Petitioners, in their motion for reconsideration, raised, inter alia, the issue on the maintenance of the 60%-40% relationship between the NIDC and KAWASAKI arising from the Constitution because PHILSECO is a landholding corporation and need not be a public utility to be bound by the 60%-40% constitutional limitation.

ISSUE: Whether or not the respondent is prohibited to possess the disputed property considering the prohibition stipulated in the 1987 Constitution against foreign owned companies.

HELD: The court upheld the validity of the mutual rights of first refusal under the JVA between KAWASAKI and NIDC. The right of first refusal is a property right of PHILSECO shareholders, KAWASAKI and NIDC, under the terms of their JVA. This right allows them to purchase the shares of their co-shareholder before they are offered to a third party. The agreement of co-shareholders to mutually grant this right to each other, by itself, does not constitute a violation of the provisions of the Constitution limiting land ownership to Filipinos and Filipino corporations. As PHILYARDS correctly puts it, if PHILSECO still owns the land, the right of first refusal can be validly assigned to a qualified Filipino entity in order to maintain the 60%-40% ration. This transfer by itself, does not amount to a violation of the Anti-Dummy Laws, absent proof of any fraudulent intent. The transfer could be made either to a nominee or such other party which the holder of the right of first refusal feels it can comfortably do business with.

Alternatively, PHILSECO may divest of its landholdings, in which case KAWASAKI, in exercising its right of first refusal, can exceed 40% of PHILSECOs equity. In fact, in can even be said that if the foreign shareholdings of a landholding corporation exceeds 40%, it is not the foreign stockholders ownership of the shares which is adversely affected but the capacity of the corporation to one land that is, the corporation becomes disqualified to own land. This finds support under the basic corporate law principle that the corporation and its stockholders are separate judicial entities. In this vein, the right of first refusal over shares pertains to the shareholders whereas the capacity to own land pertains to the corporation. Hence, the fact that PHILSECO owns land cannot deprive stockholders of their right of first refusal. No law disqualifies a person from purchasing shares in a landholding corporation even if the latter will exceed the allowed foreign equity, what the law disqualifies is the corporation from owning land.

Section 3. Lands of the Public Domain

16.

DIRECTOR OF LANDS VS. AQUINO 192 SCRA 296 (1990)

FACTS: The center of controversy in the instant petition for review on certiorari is a limestone-rich 70-hectare land in Bucay, Abra 66 hectares of which are, according to petitioners, within the Central Cordillera Forest Reserve. Private respondent Abra Industrial Corporation (AIC for brevity), a duly registered corporation established for the purpose of setting up a cement factory, claims on the other hand, to be the owner in fee simple of the whole 70-hectare area indicated in survey plans PSU-217518, PSU217519 and PSU-217520 with a total assessed value of P6,724.48. Thus, on September 23, 1965, it filed in the then Court of First Instance of Abra an application for registration in its name of said parcels of land under the Land Registration Act or, in the alternative, under Sec. 48 of Commonwealth Act No. 1411 as amended by Republic Act No.1942 inasmuch as its predecessors-in-interest had allegedly been in possession thereof since July26, 1894.

ISSUE: Whether or not lower court erred in granting the application for registration of the parcels of land notwithstanding petitioners finding that they are within the forest zone.

HELD: Yes. Forest lands or forest reserves are incapable of private appropriation and possession thereof, however long, cannot convert them into private properties. This ruling is premised on the regalian doctrine enshrined not only in the 1935 and 1973 Constitutions but also in the 1987Constitution Article XIII. The lower court closed its

eyes to a basic doctrine in land registration cases that the inclusion in a title of a part of the public domain nullifies the title. Its decision to order the registration of an inalienable land in favor of AIC under the misconception that it is imperative for the Director of Forestry to object to its exclusion from the forest reserve even in the face of its finding that indeed a sizable portion of the Central Cordillera Forest Reserve is involved, cannot be allowed to stay unrevised. It betrays an inherent infirmity which must be corrected.

17.

REPUBLIC VS. CA 160 SCRA 228 (1988)

FACTS: The owner of a piece of land has rights not only to its surface but also to everything underneath and the airspace above it up to a reasonable height. The rights over the land are indivisible and the land itself cannot be half agricultural and half mineral. The classification must be categorical; the land must be either completely mineral or completely agricultural. ISSUE: Whether respondent courts decision, i.e. the surface rights of the de la Rosas over the land while at the same time reserving the sub-surface rights of Benguet and Atok by virtue of their mining claim, is correct. HELD: No. Our holding is that Benguet and Atok have exclusive rights to the property in question by virtue of their respective mining claims which they validly acquired before the Constitution of 1935 prohibited the alienation of all lands of the public domain except agricultural lands, subject to vested rights existing at the time of its adoption. The land was not and could not have been transferred to the private respondents by virtue of acquisitive prescription, nor could its use be shared simultaneously by them and the mining companies for agricultural and mineral purposes. It is true that the subject property was considered forest land and included in the Central Cordillera Forest Reserve, but this did not impair the rights already vested in Benguet and Atok at that time. Such rights were not affected either by the stricture in the Commonwealth Constitution against the alienation of all lands of the public domain except those agricultural in nature for this was made subject to existing rights. The perfection of the mining claim converted the property to mineral land and under the laws then in force removed it from the public domain. By such act, the locators acquired exclusive rights over the land, against even the government, without need of any further act such as the purchase of the land or the obtention of a patent over it. As the land had become the private property of the locators, they had the right to transfer the same, as they did, to Benguet and Atok. The Court of Appeals justified this by saying there is no conflict of interest between the owners of the surface rights and the owners of the sub-surface rights. This is rather doctrine, for it is a well-known principle that the owner of piece of

land has rights not only to its surface but also to everything underneath and the airspace above it up to a reasonable height. Under the aforesaid ruling, the land is classified as mineral underneath and agricultural on the surface, subject to separate claims of title. This is also difficult to understand, especially in its practical application. The Court feels that the rights over the land are indivisible and that the land itself cannot be half agricultural and half mineral. The classification must be categorical; the land must be either completely mineral or completely agricultural. In the instant case, as already observed, the land which was originally classified as forest land ceased to be so and became mineral and completely mineral once the mining claims were perfected. As long as mining operations were being undertaken thereon, or underneath, it did not cease to be so and become agricultural, even if only partly so, because it was enclosed with a fence and was cultivated by those who were unlawfully occupying the surface. This is an application of the Regalian doctrine which, as its name implies, is intended for the benefit of the State, not of private persons. The rule simply reserves to the State all minerals that may be found in public and even private land devoted to agricultural, industrial, commercial, residential or (for) any purpose other than mining. Thus, if a person is the owner of agricultural land in which minerals are discovered, his ownership of such land does not give him the right to extract or utilize the said minerals without the permission of the State to which such minerals belong. The flaw in the reasoning of the respondent court is in supposing that the rights over the land could be used for both mining and non-mining purposes simultaneously. The correct interpretation is that once minerals are discovered in the land, whatever the use to which it is being devoted at the time, such use may be discontinued by the State to enable it to extract the minerals therein in the exercise of its sovereign prerogative. The land is thus converted to mineral land and may not be used by any private party, including the registered owner thereof, for any other purpose that will impede the mining operations to be undertaken therein, For the loss sustained by such owner, he is of course entitled to just compensation under the Mining Laws or in appropriate expropriation proceedings.

18.

APEX MINING VS. SOUTHEAST MINDANAO GOLD, INC., GR No. 152613, June 23, 2006

FACTS: The case involves the Diwalwal Gold Rush Area (Diwalwal), a rich tract of mineral land located inside the Agusan-Davao-Surigao Forest Reserve in Davao del Norte and Davao Oriental. Since the early 1980s, Diwalwal has been stormed by conflicts brought about by numerous mining claims over it. On March 10, 1986, Marcopper Mining Corporation (MMC) was granted an Exploration Permit(EP 133) by the Bureau of Mines and Geo-Sciences (BMG). A long battle ensued between Apex and MMC with the latter seeking the cancellation of the mining claims of Apex on the ground that such mining claims were within a forest reservation (Agusan-Davao-Surigao Forest Reserve) and thus the acquisition on mining rights should have been through an application for a permit to prospect with the BFD and not through registration of a DOL with the BMG. When it reached the SC in 1991, the Court ruled against Apex holding that the area is a forest reserve and thus it should have applied for a permit to prospect with the BFD. On February 16 1994, MMC assigned all its rights to EP 133 to Southeast Mindanao Gold Mining Corporation (SEM) a domestic corporation which is alleged to be a 100%-owned subsidiary of MMC. Subsequently, BMG registered SEMs Mineral Production Sharing Agreement (MPSA) application and the Deed of Assignment. Several oppositions were filed. The Panel of Arbitrators created by the DENR upheld the validity of EP 133. During the pendency of the case, DENR AO No. 2002-18 was issued declaring an emergency situation in the Diwalwal Gold Rush Area and ordering the stoppage of all mining operations therein.

ISSUE: Whether or not the DENR Secretary has authority to issue DAO 66 declaring 729 hectares of the areas covered by the Agusan-Davao-Surigao Forest Reserve as nonforest lands and open to small-scale mining purposes HELD:

NO. The DENR Secretary has no power to convert forest reserves into non-forest reserves. Such power is vested with the President. The DENR Secretary may only recommend to the President which forest reservations are to be withdrawn from the coverage thereof. Thus, DAO No. 66 is null and void for having been issued in excess of the DENR Secretarys authority. 19. DIRECTOR OF LANDS VS. IAC 146 SCRA 509 (1986)

FACTS: Defendant through his lawyer, filed an answer therein admitting the averment in the complaint that the land was acquired by the plaintiff through inheritance from his parents, the former owners thereof. Subsequently, the defendant changed his counsel, and with leave of court, amended the answer. In the amended answer, the admission no longer appears. The alleged ownership of the land by the plaintiff was denied coupled with an allegation that the defendant is the owner of the land as he bought it from the plaintiffs parents while they were still alive. After trial, the lower court upheld the defendants ownership of the land. On appeal, the plaintiff contended that the defendant is bound by the admission contained in his original answer.

ISSUE: Whether or not the contention of plaintiff correct HELD: NO. The original pleading had been amended such that it already disappeared from the record, lost its status as pleadings and cease to be judicial admissions. While such may be utilizedagainst the pleader as extrajudicial admissions, they must, in order to have su ch effect, be formally offered in evidence.

20.

TEN FORTY REALTY VS. LORENZANA GR No. 151212, Sept. 10, 2003

FACTS: Petitioner filed an ejectment complaint against Marina Cruz(respondent) before the MTC. Petitioner alleges that the land in dispute was purchased from Barbara Galino on December 1996, and that said land was again sold to respondent on April 1998; On the other hand, respondent answer with counterclaim that never was there an occasion when petitioner occupied a portion of the premises. In addition, respondent alleges that said land was a public land (respondent filed a miscellaneous sales application with the Community Environment and Natural Resources Office) and the action for ejectment cannot succeed where it appears that respondent had been in possession of the property prior to the petitioner; On October 2000, MTC ordered respondent to vacate the land and surrender to petitioner possession thereof. On appeal, the RTC reversed the decision. CA sustained the trial courts decision. ISSUE: Whether or not petitioner should be declared the rightful owner of the property. HELD: No. Respondent is the true owner of the land.1) The action filed by the petitioner, which was an action for unlawful detainer, is improper. As the bare allegation of petitioners tolerance of respondents occupation of the premises has not been proven, the possession should be deemed illegal from the beginning. Thus, the CA correctly ruled that the ejectment case should have been for forcible entry. However, the action had already prescribed because the complaint was filed on May 12, 1999 a month after the last day forfiling;2) The subject property had not been delivered to petitioner; hence, it did not acquire possession either materially or symbolically. As between the two buyers, therefore, respondent was first in actual possession of the property. As regards the question of whether there was good faith in the second buyer. Petitioner has not proven that respondent was aware that her mode of acquiring the

property was defective at the time she acquired it from Galino. At the time, the property which was public land had not been registered in the name of Galino; thus, respondent relied on the tax declarations thereon. As shown, the formers name appeared on the tax declarations for the property until its sale to the latter in 1998. Galino was in fact occupying the realty when respondent took over possession. Thus, there was no circumstance that could have placed the latter upon inquiry or required her to further investigate petitioners right of ownership.

21.

CHAVEZVS. PEA GR No. 133250, July 9, 2002

FACTS: In 1973, the Comissioner on Public Highways entered into a contract to reclaim areas of Manila Bay with the Construction and Development Corportion of the Philippines (CDCP). PEA (Public Estates Authority) was created by President Marcos under P.D. 1084, tasked with developing and leasing reclaimed lands. These lands were transferred to the care of PEA under P.D. 1085 as part of the Manila Cavite Road and Reclamation Project (MCRRP). CDCP and PEA entered into an agreement that all future projects under the MCRRP would be funded and owned by PEA. By 1988, President Aquino issued Special Patent No. 3517 transferring lands to PEA. It was followed by the transfer of three Titles (7309, 7311 and 7312) by the Register of Deeds of Paranaque to PEA covering the three reclaimed islands known as the FREEDOM ISLANDS. Subsquently, PEA entered into a joint venture agreement (JVA) with AMARI, a Thai-Philippine corporation to develop the Freedom Islands. Along with another 250 hectares, PEA and AMARI entered the JVA which would later transfer said lands to AMARI. This caused a stir especially when Sen. Maceda assailed the agreement, claiming that such lands were part of public domain (famously known as the mother of all scams).

Peitioner Frank J. Chavez filed case as a taxpayer praying for mandamus, a writ of preliminary injunction and a TRO against the sale of reclaimed lands by PEA to AMARI and from implementing the JVA. Following these events, under President Estradas admin, PEA and AMARI entered into an Amended JVA and Mr. Chaves claim that the contract is null and void.

ISSUE:

Whether or not the transfer to AMARI lands reclaimed or to be reclaimed as part of the stipulations in the (Amended) JVA between AMARI and PEA violate Sec. 3 Art. XII of the 1987 Constitution HELD: On the issue of Amended JVA as violating the constitution: The 157.84 hectares of reclaimed lands comprising the Freedom Islands, now covered by certificates of title in the name of PEA, are alienable lands of the public domain. PEA may lease these lands to private corporations but may not sell or transfer ownership of these lands to private corporations. PEA may only sell these lands to Philippine citizens, subject to the ownership limitations in the 1987 Constitution and existing laws.

Section 4. Specific Limits of Forest Lands and National Parks 22. LA BUGAL-B'LAAN TRIBAL ASSN. VS. DENR GR127872, Jan 27, 2004, MR GR 127882, Dec. 1, 2004

FACTS: On January 27, 2004, the Court en banc promulgated its Decision granting the Petition and declaring the unconstitutionality of certain provisions of RA 7942, DAO 9640, as well as of the entire FTAA executed between the government and WMCP, mainly on the finding that FTAAs are service contracts prohibited by the 1987 Constitution. The Decision struck down the subject FTAA for being similar to service contracts, which, though permitted under the 1973 Constitution, were subsequently denounced for being antithetical to the principle of sovereignty over our natural resources, because they allowed foreign control over the exploitation of our natural resources, to the prejudice of the Filipino nation. ISSUE: Are foreign-owned corporations in the large-scale exploration, development, and utilization of petroleum, minerals and mineral oils limited to technical or financial assistance only? HELD: Only technical assistance or financial assistance agreements may be entered into, and only for large-scale activities. Full control is not anathematic to day-to-day management by the contractor, provided that the State retains the power to direct overall strategy; and to set aside, reverse or modify plans and actions of the contractor. The idea of full control is similar to that which is exercised by the board of directors of a private corporation: the performance of managerial, operational, financial, marketing and other functions may be delegated to subordinate officers or given to contractual entities, but the board retains full residual control of the business.

Section 5. Ancestral Lands and Domain 23. CRUZ VS. SEC. OF DENR 347 SCRA 128 (2000)

FACTS: Cruz, a noted constitutionalist, assailed the validity of the RA 8371 or the Indigenous Peoples Rights Act on the ground that the law amount to an unlawful deprivation of the States ownership over lands of the public domain as well as minerals and other natural resources therein, in violation of the regalian doctrine embodied in Section 2, Article XII of the Constitution. The IPRA law basically enumerates the rights of the indigenous peoples over ancestral domains which may include natural resources. Cruz et al contend that, by providing for an all-encompassing definition of ancestral domains and ancestral lands which might even include private lands found within said areas, Sections 3(a) and 3(b) of said law violate the rights of private landowners. ISSUE: Whether or not the IPRA law is unconstitutional. HELD: The SC deliberated upon the matter. After deliberation they voted and reached a 7-7 vote. They deliberated again and the same result transpired. Since there was no majority vote, Cruzs petition was dismissed and the IPRA law was sustained. Hence, ancestral domains may include public domain somehow against the regalian doctrine.

Section 6. Common Good 24. FACTS:

TELECOM VS. COMELEC 289 SCRA 337 (1998)

Petitioner Telecommunications and Broadcast Attorneys of the Philippines, Inc. (TELEBAP) is an organization of lawyers of radio and television broadcasting companies. It was declared to be without legal standing to sue in this case as, among other reasons, it was not able to show that it was to suffer from actual or threatened injury as a result of the subject law. Petitioner GMA Network, on the other hand, had the requisite standing to bring the constitutional challenge. Petitioner operates radio and television broadcast stations in the Philippines affected by the enforcement of Section 92, B.P. No. 881. Petitioners challenge the validity of Section 92, B.P. No. 881 which provides: Comelec Time- The Commission shall procure radio and television time to be known as the Comelec Time which shall be allocated equally and impartially am ong the candidates within the area of coverage of all radio and television stations. For this purpose, the franchise of all radio broadcasting and television stations are hereby amended so as to provide radio or television time, free of charge, during the period of campaign. Petitioner contends that while Section 90 of the same law requires COMELEC to procure print space in newspapers and magazines with payment, Section 92 provides that air time shall be procured by COMELEC free of charge. Thus it contends that Section 92 singles out radio and television stations to provide free air time. Petitioner claims that it suffered losses running to several million pesos in providing COMELEC Time in connection with the 1992 presidential election and 1995 senatorial election and that it stands to suffer even more should it be required to do so again this year. Petitioners claim that the primary source of revenue of the radio and television stations is the sale of air time to advertisers and to require these stations to provide free air time is to authorize unjust taking of private property. According to petitioners, in 1992 it lost P22,498,560.00 in providing free air time for one hour each day and, in this years elections, it stands to lost P58,980,850.00 in view of COMELECs requirement that it provide at least 30 minutes of prime time daily for such.

ISSUE: Whether or not Section 92 of B.P. No. 881 denies radio and television broadcast companies the equal protection of the laws. HELD: Petitioners argument is without merit. All broadcasting, whether radio or by television stations, is licensed by the government. Airwave frequencies have to be allocated as there are more individuals who want to broadcast that there are frequencies to assign. Radio and television broadcasting companies, which are given franchises, do not own the airwaves and frequencies through which they transmit broadcast signals and images. They are merely given the temporary privilege to use them. Thus, such exercise of the privilege may reasonably be burdened with the performance by the grantee of some form of public service. In granting the privilege to operate broadcast stations and supervising radio and television stations, the state spends considerable public funds in licensing and supervising them.

Section 7. Private Lands 25.

REPUBLIC VS. CA 235 SCRA 567

FACTS:

Respondent Angelina M. Castro and Edwin F. Cardenas were married in a civil ceremony performed by a City Court Judge of Pasig City and was celebrated without the knowledge of Castro's parents. Defendant Cardenas personally attended the procuring of the documents required for the celebration of the marriage, including the procurement of the marriage license. The couple did not immediately live together as husband and wife since the marriage was unknown to Castro's parents. They decided to live together when Castro discovered she was pregnant. The cohabitation lasted only for four months. Thereafter, the couple parted ways. Desiring to follow her daughter in the U.S, Castro wanted to put in order he marital status before leaving for the U.S. She then discovered that there was no marriage license issued to Cardenas prior to the celebration of their marriage as certified by the Civil Registrar of Pasig, Metro Manila. Respondent then filed a petition with the RTC of Quezon City seeking for the judicial declaration of nullity of her marriage claiming that no marriage license was ever issued to them prior to the solemnization of their marriage. The trial court denied the petition holding that the certification was inadequate to establish the alleged non-issuance of a marriage license prior to the celebration of the marriage between the parties. It ruled that the "inability of the certifying official to locate the marriage license is not conclusive to show that there was no marriage license issued. On appeal, the decision of the trial court was reversed.

ISSUE:

Is the marriage valid? Is there such a thing as a "secret marriage"?

HELD:

At the time of the subject marriage was solemnized on June 24, 1970, the law governing marital relations was the New Civil Code. The law provides that no marriage license shall be solemnized without a marriage license first issued by the local civil registrar. Being one of the essential requisites of a valid marriage, absence of a license would render the marriage void ab initio. It will be remembered that the subject marriage was a civil ceremony performed by a judge of a city court. The subject marriage is one of those commonly known as a "secret marriage" - a legally non-existent phrase but ordinarily used to refer to a civil marriage celebrated without the knowledge of the relatives and/or friends of either or both of the contracting parties. The records show that the marriage between Castro and Cardenas as initially unknown to the parents of the former.

26.

ZARAGOSA VS. CA GR No. 106401, September 29, 2000

FACTS: Flavio Zaragoza Cano was the registered owner of certain parcels of land. He had four children namely: Gloria, Zacariaz, Florentino and Alberta, all surnamed Zaragoza. On December 9, 1964, he died without a will and was survived by his four children. On December 28, 1981, private respondent Alberta Zaragoza Morgan filed a complaint with the Court of First Instance against petitioner-spouses Florentino and Erlinda for delivery of her inheritance share and for payment of damages. She claims that she is a natural-born Filipino citizen and the youngest child of the late Flavio. She further alleged that her father in his lifetime partitioned the properties among his four children. The shares of her brothers and sister were given to them in advance by way of deed of sale, but without valid consideration, while her share was not conveyed by way of deed of sale then. Petitioners denied knowledge of an alleged distribution by way of deeds of sale to them by their father. They denied knowledge of the alleged intention of their father to convey the cited lots to Alberta, much more, the reason for his failure to do so because she became an American citizen. They denied that there was partitioning of the estate of their father during his lifetime. The Regional Trial Court rendered judgment adjudicating Lot 471 in the name of Flavio Zaragoza Cano to Alberta Zaragoza-Morganas appertaining her share in his estate. ISSUE: Whether the partition inter vivos by Flavio Zaragoza Cano of his properties is valid. HELD: Both the trial court and the public respondent found that during the lifetime of Flavio, healready partitioned and distributed hisproperties among his three children,excepting private respondent through deeds of sale. A deed of sale was not executed in favor of private respondent because she had become an American citizen and the Constitution prohibited a sale in her favor.It is basic in the law of succession that apartiti on inter vivos may be done for as long as legitimes are not prejudiced. Unfortunately, collation cannot be done inthis case where the original petition fordelivery of inheritance share only impleaded one of the other compulsory heirs. The petition must therefore be

dismissed without prejudice to the institution of a new proceeding where all the indispensable parties are present for the rightful determination of their respective legitime and if the legitimes were prejudiced by the partitioning inter vivos. Private respondent in submitting her petition for the delivery of inheritance share, was in effect questioning the validity of the deed of sale in favor of petitioner and consequently, the transfer of certificate of title issued in the latters name. although the trial court , as an orbiter, made a finding of validity of the conveyance of the said lot, since according toit, private respondent did not question thegenuineness of the signature of thedecease d, nevertheless, when the case was elevated to the Court of Appeals, the latter declared the sale to be fictitious because of finding of marked differences in the signature of Flavio in the deed of sale vis--vissig natures found in earlier documents. Could this be done? The petition is a collateralattack. A certificate of title shall not besubject to collateral attack. It cannot bealtered, modified or cancelled except in adirect proceeding in accordance with law.

27.

RAMIREZ VS. VDA. DE RAMIREZ 111 SCRA 704 (1982)

FACTS: The deceased was survived by his spouse, 2 grandnephews, and his companion. The administrator submitted a partition to the court which divided the estate into 2: onehalf would go to the widow in satisfaction of her legitime; the other half, which is the free portion, would go to the grandnephews; however, 1/3 of the free portion is charged with the widows usufruct and the remaining 2/3 with a usufruct in favor of the companion. The grandnephews opposed the substitution on the ground that the 1 st heirs are not related to the substitutes within the 1st degree. ISSUE: Whether the fideicommissary substitution is valid if the substitutes are related to the companion within one degree. HELD: SC ruled that the fideicommissary substitution is void. The substitutes (grandnephews) are not related to the companion within one degree. In effect, the SC ruled that one degree means one generation and not one designation. So, it follows that the fideicommissary can only be either a child or a parent of the 1 st heir.

28.

HALILI VS. CA 287 SCRA 465 (1998)

FACTS: Federico Suntay was a gubernatorial candidate in Bulacan in the 1951 elections. Fortunato F. Halili, was the incumbent governor, the Liberal Party head, Suntays campaign manager, and was also a public utility operator. Suntay needed funds to finance his campaign. Halili agreed to make cash advances to Suntay. The parties encountered certain obstacles brought about by Articles 47, 48, 183, 184 and 185 of the Revised Election Code. Section 47 (Unlawful contributions) provides that it shall be unlawful for any corporation or entity operating a public utility or which is in possession of or is exploiting any natural resources of the nation to contribute or make any expenditure in connection with any election campaign. On the other hand, Section 48 (Limitation upon expenses of candidates) provides that No candidate shall spend for his election campaign more than the total amount of the emoluments for one year attached to the office for which he is a candidate. To go around the law, a scheme was hatched for the concealment or for the laundering of the loans and advances of Halili for Suntays campaign; and to implement, the advances or loans were made in the names of Halilis trusted employees as dummies. Suntay leased his fishpond to Halilis employees for a four-year period, with stipulated rental of P8,000 per year. Certain promissory notes were made during the transactions. After the expiration of the lease contract, Suntay filed a case against Halili and others, praying that the notes be declared void pursuant to Article 1409 of the Civil Code for lack of consideration and for being contrary to Section 47 and 48 of the Election Law. ISSUE: Whether Section 47 applies to a natural person and whether Section 48 applies to a non-candidate. HELD: If a corporation operating a public utility is prohibited from making a political contribution or expenditure, there is no valid reason for not applying the prohibition (Section 47) to a natural person operating a public service business. Furthermore, Section 48 applies to a non-candidate like Halili because Section 184 of the Revised Election Code (Persons criminally responsible for election offenses) speaks of principals and accomplices. Halili was no ordinary lender and lessee as knew that the rental and the loans would be spent for Suntays candidacy. He was not only Suntays financial

backer but, as campaign manager, he had a hand in the expenditure of the funds supplied by him to Suntay. He was Suntays co-principal. 29. LEE VS. REPUBLIC 366 SCRA (2001)

FACTS: Appellant notified Appellees that they were in default on their second mortgage and the principal balance was being accelerated. Appellees sent Appellant a certified check for the two overdue payments and the next payment due. Appellant returned the check and filed suit to foreclose. Appellees moved to reinstate the mortgage. The court granted this motion finding that there was substantial equity in the property, the first mortgage was current, and the second mortgage would be paid from the proceeds of the Appellees proposed sale of the property.

ISSUE: Whether the mortgagor had the right to prevent the mortgage holder from exercising his option to accelerate by tendering the arrears due. HELD: No. This right terminates once the mortgage holder has exercised his option to accelerate. The mortgagor upon the occurrence of a default may tender the arrears due and thereby prevent the mortgage holder form exercising his option to accelerate. However, once the mortgage holder has exercised his option to accelerate, the right of the mortgagor to tender only the arrears is terminated.

30. FACTS:

FRENZEL VS. CATITO GR No. 143958, July 11, 2003

Petitioner Alfred Fritz Frenzel is an Australian citizen of German descent. He worked as a pilot with the New Guinea Airlines. He arrived in the Philippines in 1974, started engaging in business in the country two years thereafter, and married Teresita Santos, a Filipino citizen. In 1981, Alfred and Teresita separated from bed and board without obtaining a divorce. Sometime in February 1983, Alfred arrived in Sydney, Australia for a vacation. He went to King's Cross, a night spot in Sydney, for a massage where he met Ederlina Catito, a Filipina. Unknown to Alfred, she resided for a time in Germany and was married to Klaus Muller, a German national. She left Germany and tried her luck in Sydney, Australia, where she found employment as a masseuse in the King's Cross nightclub. Alfred was so enamored with Ederlina that he persuaded her to stop working at King's Cross, return to the Philippines, and engage in a wholesome business of her own. He also proposed that they meet in Manila, to which she assented. Within two weeks of Ederlina's arrival in Manila, Alfred joined her. Alfred reiterated his proposal for Ederlina to stay in the Philippines and engage in business, even offering to finance her business venture. Alfred told Ederlina that he was married but that he was eager to divorce his wife in Australia. Alfred proposed marriage to Ederlina, but she replied that they should wait a little bit longer. Alfred decided to stay in the Philippines for good and live with Ederlina. On different occasions, Alfred bought several properties in the Philippines for Ederlinas business and for the couples residence using his own funds but since Alfred knew that as an alien he was disqualified from owning lands in the Philippines, he agreed that only Ederlina's name would appear in the deed of sale as the buyer of the property, as well as in the title covering the same. Alfred also sold his properties in Australia and the proceeds of the sale were deposited in Alfred's account with the Hong Kong Shanghai Banking Corporation (HSBC), Kowloon Branch. When Ederlina opened her own account with HSBC Kowloon, Alfred transferred his with the said bank to this new account. Alfred received a Letter from Klaus Muller. Klaus informed Alfred that he and Ederlina had been married on October 16, 1978 and had a blissful married life until

Alfred intruded therein. Klaus stated that he knew of Alfred and Ederlina's amorous relationship and begged Alfred to leave Ederlina alone and to return her to him. When Alfred confronted Ederlina, she admitted that she and Klaus were, indeed, married. But she assured Alfred that she would divorce Klaus. Alfred was appeased. He agreed to continue the amorous relationship and wait for the outcome of Ederlina's petition for divorce. Ederlina's petition for divorce was denied twice because Klaus opposed the same. Klaus wanted half of all the properties owned by Ederlina in the Philippines before he would agree to a divorce. Worse, Klaus threatened to file a bigamy case against Ederlina. Alfred and Ederlina's relationship started deteriorating. Ederlina had not been able to secure a divorce from Klaus. The latter could charge her for bigamy and could even involve Alfred, who himself was still married. To avoid complications, Alfred decided to live separately from Ederlina and cut off all contacts with her. Ederlina complained that he had ruined her life. Shortly thereafter, Alfred filed a Complaint against Ederlina, with the RTC of Quezon City for recovery of real and personal properties. In his complaint, Alfred alleged that Ederlina, without his knowledge and consent, managed to transfer funds from their joint account in HSBC Hong Kong, to her own account with the same bank. Using the said funds, Ederlina was able to purchase the properties subject of the complaints. Ederlina failed to file her answer and was declared in default. In the meantime, Alfred also filed a complaint against Ederlina with the RTC of Davao City for specific performance, declaration of ownership of real and personal properties, sum of money, and damages. He alleged that during the period of their common-law relationship, he acquired solely through his own efforts and resources real and personal properties in the Philippines valued more or less at P724,000.00 The RTC of Quezon City ruled in favor of Alfred. However, after due proceedings in the RTC of Davao City, the trial court ruled in favor of Erlinda. The trial court ruled that based on documentary evidence, the purchaser of the three parcels of land subject of the complaint was Ederlina. The court further stated that even if Alfred was the buyer of the properties; he had no cause of action against Ederlina for the recovery of the same because as an alien, he was disqualified from acquiring and owning lands in the Philippines. The sale of the three parcels of land to the petitioner was null and void ab initio. Applying the pari delicto doctrine, the petitioner was precluded from recovering the properties from the respondent. The CA rendered a decision affirming in toto the decision of the RTC Hence, the petition at bar.

ISSUE: Can petitioner is entitled to recover the property under Article 1416 of the Civil Code? HELD: NO. Under Article 1416 of the Civil Code: When the agreement is not illegal per se but is merely prohibited, and the prohibition by the law is designed for the protection of the plaintiff, he may, if public policy is thereby enhanced, recover what he has paid or delivered. The provision applies only to those contracts which are merely prohibited, in order to benefit private interests. It does not apply to contracts void ab initio. The sale of three parcels of land in favor of the petitioner who is a foreigner is illegal per se. The transactions are void ab initio because they were entered into in violation of the Constitution. Thus, to allow the petitioner to recover the properties or the money used in the purchase of the parcels of land would be subversive of public policy.

31.

MULLER VS. MULLER GR No. 149615, August 29, 2006

FACTS: Elena and Helmut,a German national were married in Germany and resided there in a house owned by Helmuts parents but later permanently resided in the Philippines. Helmut had inherited the house in Germany from his parents which he sold and used the proceeds for the purchase of a parcel of land in Antipolo and in the construction of a house. The Antipolo property was registered in the name of Elena. After they separated, Helmut filed a motion for separation of properties for reimbursement of the property in Antipolo.

ISSUE: Whether or not respondent is entitled to reimbursement of the funds used for the acquisition of the Antipolo property?

HELD: NO. Save for the exception provided in cases of hereditary succession, Helmuts disqualification from owning lands in the Philippines is absolute. Where the purchase is made in violation of an existing statute and in evasion of its express provision, no trust can result in favor of the party who is guilty of the fraud. Helmut cannot seek reimbursement on the ground of equity where it is clear that he willingly and knowingly bought the property despite the constitutional prohibition.

32.

MATTHEWS VS. TAYLOR SPOUSES GR No. 164584, June 22, 2009

FACTS: On June 30, 1988, respondent Benjamin A. Taylor (Benjamin), a British subject, married Joselyn C. Taylor (Joselyn), a 17-year old Filipina. On June 9, 1989, Joselyn bought from Diosa M. Martin a 1,294 square-meter lot (Boracay property. The sale was allegedly financed by Benjamin. However, Benjamin and Joselyn had a falling out, and Joselyn ran away with Kim Philippsen. On June 8, 1992, Joselyn executed a Special Power of Attorney (SPA) in favor of Benjamin, authorizing the latter to maintain, sell, lease, and sub-lease and otherwise enter into contract with third parties with respect to their Boracay property. On July 20, 1992, Joselyn entered into an Agreement of Lease (Agreement) involving the Boracay property for a period of 25 years. Claiming that the Agreement was null and void since it was entered without his (Benjamins) consent, he instituted an action for Declaration of Nullity of Agreement of Lease with Damages against Joselyn and the petitioner. ISSUE: Can an alien husband nullify a lease contract entered into by his Filipina wife over a land bought during their marriage? HELD: Benjamin has no right to nullify the Agreement of Lease between Joselyn and petitioner. Benjamin, being an alien, is absolutely prohibited from acquiring private and public lands in the Philippines. Considering that Joselyn appeared to be the designated vendee in the Deed of Sale of said property, she acquired sole ownership thereto. This is true even if Benjamins claim that he provided the funds for such acqu isition is sustained. By entering into such contract knowing that it was illegal, no implied trust was created in his favor; no reimbursement for his expenses can be allowed; and no declaration can be made that the subject property was part of the conjugal/community property of the spouses.

33.

HULST VS. PR BUILDERS G.R . No. 156364 (Sept. 25, 2008)

FACTS: Jacobus Bernhard Hulst (petitioner) and his spouse Ida Johanna Hulst-Van Ijzeren (Ida), Dutch nationals, entered into a Contract to Sell with PR Builders, Inc. (respondent), for the purchase of a 210-sq m residential unit in respondent's townhouse project in Barangay Niyugan, Laurel, Batangas. When respondent failed to comply with its verbal promise to complete the project by June 1995, the spouses Hulst filed before the Housing and Land Use Regulatory Board (HLURB) a complaint for rescission of contract with interest, damages and attorney's fees, ISSUE: Whether or not the contract to purchase is valid. HELD: No. Petitioner and his wife are foreign nationals who are disqualified under the Constitution from owning real property in their names. Section 7 of Article XII of the 1987 Constitution provides: Sec. 7. Save in cases of hereditary succession, no private lands shall be transferred or conveyed except to individuals, corporations, or associations qualified to acquire or hold lands of the public domain.

34.

REPUBLIC V. CA 235 SCRA 567 [1994]

FACTS: In 1970, the spouses Juan and Juana de la Cruz, then Filipinos, bought the parcel of unregistered land in the Philippines on which they built a house which became their residence. In 1986, they migrated to Canada and became Canadian citizens. Thereafter, in 1990, they applied for the registration of the aforesaid land in their names which was opposed by the Republic. ISSUE: Should the application of the spouses de la Cruz be granted over the Republics opposition? HELD: Yes, the application should be granted. As a rule, the Constitution prohibits aliens from owning private lands in the Philippines. This rule, however, does not apply to the spouses Juan and Juana de la Cruz because at the time they acquired ownership over the land, albeit imperfect, they were still Filipino citizens. The application for registration is a mere confirmation of the imperfect title which the spouses have already acquired before they became Canadian citizens.

35.

MANILA PRINCE HOTEL VS. GSIS 267 SCRA 408

FACTS: The controversy arose when respondent Government Service Insurance System (GSIS), pursuant to the privatization program of the Philippine Government under Proclamation No. 50 dated 8 December 1986, decided to sell through public bidding 30% to 51% of the issued and outstanding shares of respondent Manila Hotel Corporation. In a close bidding held on 18 September 1995 only two (2) bidders participated: petitioner Manila Prince Hotel Corporation, a Filipino corporation, which offered to buy 51% of the MHC or 15,300,000 shares at P41.58 per share, and Renong Berhad, a Malaysian firm, with ITT-Sheraton as its hotel operator, which bid for the same number of shares at P44.00 per share, or P2.42 more than the bid of petitioner. Pending the declaration of Renong Berhad as the winning bidder/strategic partner and the execution of the necessary contracts, matched the bid price of P44.00 per share tendered by Renong Berhad. On 17 October 1995, perhaps apprehensive that respondent GSIS has disregarded the tender of the matching bid and that the sale of 51% of the MHC may be hastened by respondent GSIS and consummated with Renong Berhad, petitioner came to this Court on prohibition and mandamus. In the main, petitioner invokes Sec. 10, second par., Art. XII, of the 1987 Constitution and submits that the Manila Hotel has been identified with the Filipino nation and has practically become a historical monument which reflects the vibrancy of Philippine heritage and culture. It is a proud legacy of an earlier generation of Filipinos who believed in the nobility and sacredness of independence and its power and capacity to release the full potential of the Filipino people. To all intents and purposes, it has become a part of the national patrimony. 6 Petitioner also argues that since 51% of the shares of the MHC carries with it the ownership of the business of the hotel which is owned by respondent GSIS, a government-owned and controlled corporation, the hotel business of respondent GSIS being a part of the tourism industry is unquestionably a part of the national economy. ISSUE: Whether or Not the sale of Manila Hotel to Renong Berhad is violative of the Constitutional provision of Filipino First policy and is therefore null and void. HELD: The Manila Hotel or, for that matter, 51% of the MHC, is not just any commodity to be sold to the highest bidder solely for the sake of privatization. The Manila Hotel has

played and continues to play a significant role as an authentic repository of twentieth century Philippine history and culture. This is the plain and simple meaning of the Filipino First Policy provision of the Philippine Constitution. And this Court, heeding the clarion call of the Constitution and accepting the duty of being the elderly watchman of the nation, will continue to respect and protect the sanctity of the Constitution. It was thus ordered that GSIS accepts the matching bid of petitioner MANILA PRINCE HOTEL CORPORATION to purchase the subject 51% of the shares of the Manila Hotel Corporation at P44.00 per share and thereafter to execute the necessary clearances and to do such other acts and deeds as may be necessary for purpose. The Supreme Court directed the GSIS and other respondents to cease and desist from selling the 51% shares of the MHC to the Malaysian firm Renong Berhad, and instead to accept the matching bid of the petitioner Manila Prince Hotel. According to Justice Bellosillo, ponente of the case at bar, Section 10, second paragraph, Article 11 of the 1987 Constitution is a mandatory provision, a positive command which is complete in itself and needs no further guidelines or implementing laws to enforce it. The Court En Banc emphasized that qualified Filipinos shall be preferred over foreigners, as mandated by the provision in question. The Manila Hotel had long been a landmark, therefore, making the 51% of the equity of said hotel to fall within the purview of the constitutional shelter for it emprises the majority and controlling stock. The Court also reiterated how much of national pride will vanish if the nations cultural heritage will fall on the hands of foreigners. In his dissenting opinion, Justice Puno said that the provision in question should be interpreted as pro-Filipino and, at the same time, not anti-alien in itself because it does not prohibit the State from granting rights, privileges and concessions to foreigners in the absence of qualified Filipinos. He also argued that the petitioner is estopped from assailing the winning bid of Renong Berhad because the former knew the rules of the bidding and that the foreigners are qualified, too.

36.

ARMY AND NAVY CLUB OF MANILA, INC. VS. COURT OF APPEALS 271 SCRA 36 April 08, 1997

FACTS: Actions; Ejectment; Illegal Detainer; For violations of the lease contract and after several demands, the lessor has no other recourse but to file an action for illegal detainer and demand the lessees eviction from the premises. Petitioner failed to pay the rents for seven (7) consecutive years. As of October, 1989 when the action was filed, rental arrears ballooned to P7.2 million. Real estate taxes on the land accumulated to P6,551,408.28 as of May, 1971. Moreover, petitioner failed to erect a multi-storey hotel in the site. For violations of the lease contract and after several demands, the City of Manila had no other recourse but to file the action for illegal detainer and demand petitioners eviction from the premises. National Patrimony; Historical Markers; Statutes; R.A. 4846; Due Process; When the classification of property into historical treasures or landmarks will involve the imposition of limits on ownership, the Bill of Rights demands that it be done with due process both substantive and procedural. While the declaration that it is a historical landmark is not objectionable, the recognition is, however, specious. We take the occasion to elucidate on the views of Fr. Joaquin Bernas who was invited as amicus curiae in the recent case of Manila Prince Hotel v. GSIS where the historical character of Manila Hotel was also dealt with. He stated that: The countrys artistic and historic wealth is therefore a proper subject for the exercise of police power: . . . which the State may regulate. Thi s is a function of the legislature. And once regulation comes in, due process also comes into play. When the classification of property into historical treasures or landmarks will involve the imposition of limits on ownership, the Bill of Rights demands that it be done with due process both substantive and procedural. In recognition of this constitutional principle, the State in fact has promulgated laws, both general and special, on the subject the current general law on the subject is R.A. 4846, approved on June 18, 1966, and amended byP.D. No. 374.Nowhere in R.A. 4846 does it state that the recognition of a historical landmark grants possessory right over the property to a lessee, and neither is the National Historical Commission given the authority to vest such right of ownership or possession of a private property to another. It behooves us to think why the declaration was conferred only in 1992, three (3) years after the action for ejectment was instituted. We can only surmise that this was merely an afterthought, an attempt to thwart any legal action taken against the petitioner. Nonetheless, such certification does not give any authority to the petitioner to lay claim of ownership, or any right over the subject

property. Nowhere in the law does it state that such recognition grants possessory rights over the property to the petitioner. Nor is the National Historical Commission given the authority to vest such right of ownership or possession of private property to the petitioner. The law merely states that it shall be the policy of state to preserve and protect the important cultural properties and National Cultural Treasures of the nation and to safeguard their intrinsic value. In line with this, any restoration, reconstruction or preservation of historical buildings shall only be made under the supervision of the Director of the National Museum. The authority of the National Historical Commission is limited only to the supervision of any reconstruction, restoration or preservation of the architectural design of the identified historical building and nothing more. Even if the recognition of the Army and Navy Club as a historical landmark made by the National Historical Commission is valid, the historical significance of the Club, if any, shall not be affected if the lessees eviction from the premises is warranted. Even assuming that such recognition made by the National Historical Commission is valid, the historical significance of the Club, if any, shall not be affected if petitioners eviction from the premises is warranted. Unfortunately, petitioner is merely a lessee of the property. By virtue of the lease contract, petitioner had obligations to fulfill. Petitioner cannot just hide behind some recognition bestowed upon it in order to escape from its obligation or remain in possession. It violated the terms and conditions of the lease contract. Thus, petitioners eviction from the premises is inevitable. reconstruction, restoration or preservation of the architectural design of the identified historical building and nothing more. Even if the recognition of the Army and Navy Club as a historical landmark made by the National Historical Commission is valid, the historical significance of the Club, if any, shall not be affected if the Lessees eviction from the premises is warranted. Even assuming that such recognition made by the National Historical Commission is valid, the historical significance of the Club, if any, shall not be affected if petitioners eviction from the premises is warranted. Unfortunately, petitioner is merely a lessee of the property. By virtue of the lease contract, petitioner had obligations to fulfill. Petitioner cannot just hide behind some recognition bestowed upon it in order to escape from its obligation or remain in possession. It violated the terms and conditions of the lease contract. Thus, petitioners eviction from the premises is inevitable.

37.

TANADA V. ANGARA 272 SCRA 18 (1997)

FACTS: - April 15, 1994: Respondent Rizalino Navarro, then Sec. of the Department of Trade and Industry, representing the Government of the Republic of the Philippines, signed in Marrakesh, Morocco, the Final Act Embodying the Results of the Uruguay Round of Multilateral Negotiations (Final Act). - On December 14, 1994, the Philippine Senate adopted Resolution No. 97 which Resolved, as it is hereby resolved, that the Senate concur, as it hereby concurs, in the ratification by the President of the Philippines (Fidel V. Ramos) of the Agreement Establishing the World Trade Organization. - The WTO Agreement ratified by the President of the Philippines is composed of the Agreement Proper and the associated legal instruments included in Annexes one (1), two (2) and three (3) of that Agreement which are integral parts thereof. - The Final Act signed by Secretary Navarro embodies not only the WTO Agreement (and its integral annexes aforementioned) but also (1) the Ministerial Declarations and Decisions and (2) the Understanding on Commitments in Financial Services. - Instant petition was filed: nullification of the Philippine ratification of the WTO Agreement. ISSUE: Whether or not the provisions of the Agreement Establishing the World Trade Organization and the Agreements and Associated Legal Instruments included in Annexes one (1), two (2) and three (3) of that agreement cited by petitioners directly contravene or undermine the letter, spirit and intent of Section 19, Article II and Sections 10 and 12, Article XII of the 1987 Constitution. Sec. 19. The State shall develop a self-reliant and independent national economy effectively controlled by Filipinos. Sec. 10. The Congress shall enact measures that will encourage the formation and operation of enterprises whose capital is wholly owned by Filipinos. In the grant of rights, privileges, and concessions covering the national economy and patrimony, the State shall give preference to qualified Filipinos. Sec. 12. The State shall promote the preferential use of Filipino labor, domestic materials and locally produced goods, and adopt measures that help make them competitive. HELD: The question thus posed is judicial rather than political. The duty (to adjudicate) remains to assure that the supremacy of the Constitution is upheld. Once a controversy as to the application or interpretation of a constitutional provision is raised before this Court (as in the instant case), it becomes a legal issue which the Court is bound by constitutional mandate to decide.

It is true that in the recent case of Manila Prince Hotel vs. Government Service Insurance System, et al., this Court held that Sec. 10, second par., Art. XII of the 1987 Constitution is a mandatory, positive command which is complete in itself and which needs no further guidelines or implementing laws or rules for its enforcement. From its very words the provision does not require any legislation to put it in operation. It is per se judicially enforceable. However, as the constitutional provision itself states, it is enforceable only in regard to the grants of rights, privileges and concessions covering national economy and patrimony and not to every aspect of trade and commerce. It is not difficult to answer this question. Constitutions are designed to meet not only the vagaries of contemporary events. They should be interpreted to cover even future and unknown circumstances. In rendering this Decision, this Court never forgets that the Senate, whose act is under review, is one of two sovereign houses of Congress and is thus entitled to great respect in its actions. It is itself a constitutional body independent and coordinate, and thus its actions are presumed regular and done in good faith. Unless convincing proof and persuasive arguments are presented to overthrow such presumptions, this Court will resolve every doubt in its favor. Using the foregoing well-accepted definition of grave abuse of discretion and the presumption of regularity in the Senates processes, this Court cannot find any cogent reason to impute grave abuse of discretion to the Senates exercise of its power of concurrence in the WTO Agreement granted it by Sec. 21 of Article VII of the Constitution. That the Senate, after deliberation and voting, voluntarily and overwhelmingly gave its consent to the WTO Agreement thereby making it a part of the law of the land is a legitimate exercise of its sovereign duty and power. What the Senate did was a valid exercise of its authority. As to whether such exercise was wise, beneficial or viable is outside the realm of judicial inquiry and review. That is a matter between the elected policy makers and the people. The petition was dismissed for lack of merit.

38.

B A G A T S I N G V S COMMITTEE ON PRIVATIZATION

FACTS: PETRON was originally registered with the Securities and Exchange Commission (SEC) in 1966 under the corporate name "Esso Philippines, Inc." .ESSO became a wholly-owned company of the government under the corporate name PETRON and as a subsidiary of PNOC.P E T R O N o w n s t h e l a r g e s t , m o s t m o d e r n c o m p l e x r e f i n e r y in the Philippines. It is listed as the N o . 1 c o r p o r a t i o n i n t e r m s o f a s s e t s a n d i n c o m e i n t h e Philippines in 1993.President Corazon C. Aquino promulgated Proclamation No. 50 in the exercise of her legislative power under the Freedom Constitution. Implicit in the Proclamation is the need to raise revenue for the Government and the ideal of leaving business to the private sector by creating the committee on privatization. The Government can then concentrate on the delivery of basic services and the performance e of vital public functions. The Presidential Cabinet of President Ramos approved t h e p r i v a t i z a t i o n o f P E T R O N a s p a r t o f t h e E n e r g y Sector Action Plan. PNOC Board of Directors passed a resolution authorizing the company to negotiate and conclude a contract with the consortium of Salomon Brothers of Hongkong Limited and PCI Capital Corporation for financial advisory services to be rendered to PETRON. The Petron Privatization Working Committee (PWC) was thus formed. It finalized a privatization strategy with 40% of the shares to be sold to a strategic partner and 20% to the general public T h e P r e s i d e n t a p p r o v e d t h e 4 0 % 4 0 % 2 0 % privatization strategy of PETRON. T h e i n v i t a t i o n t o b i d w a s p u b l i s h e d i n s e v e r a l n e w s p a p e r s o f g e n e r a l c i r c u l a t i o n , b o t h l o c a l a n d foreign. The PNOC Board of Directors then passed Resolution No. 866, S. 1993, declaring ARAMCO the winning bidder. PNOC and ARAMCO signed the Stock Purchase Agreement, the two companies signed the Shareholders' Agreement. The petition for prohibition in G.R. No. 112399 sought:(1) to nullify the bidding conducted for the sale of a block of shares constituting 40% of the capital stock(40% block) of Petron Corporation (PETRON) and the award made to Aramco Overseas Company, B.

V .(ARAMCO) as the highest bidder and (2) to stop the sale of said block of shares to ARAMCO. The petition for prohibition and Certiorari in G.R. No. 115994 sought to annul the sale of the s a m e b l o c k o f P e t r o n s h a r e s subject of the petition in G.R. No. 112399.ARAMCO entered a limited appearance to question the jurisdiction over its person, alleging that it is a foreign company organized under the laws of the Netherlands, that it is not doing nor licensed to do business in the Philippines, and that it does not maintain an office or a business address in and has not appointed a resident agent for the Philippines Petitioners however, countered that they filed the capacity as members of Congress. action in their

ISSUE: WON Petitioners have a locus standi

HELD: Petition is dismissed.In Philippine Constitution Association v.Hon. Salvador Enriquez , G.R. No. 113105, August 19, 1994, we held that the members of Congress have the legal standing to question the validity of acts of the Executive which i n j u r e s t h e m i n t h e i r p e r s o n o r t h e i n s t i t u t i o n o f Congress to which they belong. In the latter case, the acts cause derivative but nonetheless substantial i n j u r y w h i c h c a n b e q u e s t i o n e d b y m e m b e r s o f Congress (Kennedy v. James, 412 F. Supp. 353 [1976]).In the absence of a claim that the contract in question v i o l a t e d t h e r i g h t s o f p e t i t i o n e r s o r i m p e r m i s s i b l y intruded into the domain of the Legislature, petitioners have no legal standing to institute the instant action in their capacity as members of Congress. However, petitioners can bring the action in their capacity as taxpayers under the doctrine laid down in Kilosbayan, Inc. v Guingona , 232 SCRA 110 (1994).Under said ruling, taxpayers may question contracts entered into by the national government or governmentowned or controlled corporations alleged to be in contravention of the law. As long as the ruling in Kilosbayan on locus standi is not reversed, we have no choice but to follow it and uphold the legal standing o f p e t i t i o n e r s a s t a x p a y e r s t o i n s t i t u t e t h e p r e s e n t action.

PRIVATIZATION The only requirement under R.A. No. 7181 in order to privatize a strategic industry like PETRON is the approval o f t h e P r e s i d e n t . I n t h e c a s e o f P E T R O N ' s privatization, the President gave his approval not only once but twice. PETRON's privatization is also in line with and is part of the Philippine Energy Program under R.A. No. 7638.Section 5(b) of the law provides that the Philippine Energy Program shall include a policy direction towards the privatization of government agencies related to energy

39.

Albano vs. Reyes (175 SCRA 264)

FACTS:

PPA ( Philippine Ports Authority ) Board adopted its Resolution No. 850 directing PPA management to prepare the Invitation to Bid and all relevant documents and technical requirements necessary for the public bidding of the development, management and operation for the MICT ( leasing as well as to implement this project. Respondent Secretary Reyes created a 7 man Special MICT Bidding Committee charged with all bid proposals. After evaluation of the seven companies that submitted bids, the committee recommended the award of the contract to ICTSI for having offered the best technical and financial proposal. However, before the MICT contract could be signed, 2 cases were filed against respondents which assailed the legality and regularity of the bidding. But on May 18, 1988, the President of the Philippines approved the proposed MICT Contract with specific directives on the part of the PPA and the contractor ICTSI. Meanwhile, Rodolfo Albano, the petitioner filed a petition assailing the award of the MICT contract to ICTSI claiming that the former is a public utility and therefore needs a legislative franchise before it can legally operate as a public utility, pursuant to Article 12, Sec 11 of the 1987 Constitution.

Issue: Whether or not the MICT needs a legislative franchise from Congress to legally operate as a public utility?

Held: NO. EO No. 30 dated July16, 1986 provides for the immediate recall of the franchise granted to the Manila International Port Terminals Inc., and authorize the PPA to take over, manage and operate the Manila International Port Complex at North Harbor, Manila and undertake the provision of cargo handling and port related services thereat, in accordance with PD 857 and other applicable laws and regulations. Sec. 6 of PD 857 otherwise known as the Revised Charter of the PPA provides as one of the corporate duties of the PPA is to provide services ( whether on its own, by contract, or otherwise ) within the Port Districts and the approaches thereof including but not limited to

As stated above, PPA has been tasked under EO No. 30, with the management and operation of the Manila International Port Complex in accordance with PD 857 and other applicable laws and regulations. However, PD 857 itself authorizes the PPA to perform the service by itself, by contracting it out, or through other means. Reading EO No. 30 and PD 857 together, the inescapable conclusion is that the lawmaker has empowered the PPA to undertake by itself the operation and management of the MICP or to authorize its operation and management by another by contract or other means at its option. The latter power having been delegated to the PPA, a franchise from Congress to authorize an entity other than the PPA to operate and manage the MICP becomes unnecessary. Therefore, PPAs act of privatizing the MICT and awarding the Contract to ICTSI are wholly within its jurisdiction under its Charter which empowers the PPA to supervise, control, regulate, construct, maintain, operate and provide such facilities necessary in the ports vested

40.

Tatad vs. Garcia 241 SCRA 334, GR. No. 114222. April 6, 1995

FACTS: DOTC planned to construct a light railway transit line along Edsa. EDSA LRT Corporation, Ltd., a foreign corporation was awarded the contract to build, lease and transfer the said light railway. The said award was questioned by the petitioners on the basis that a foreign corporation cannot own the EDSA LRT III, a public utility as it violates the Constitution.

ISSUE: Whether or not an owner and lessor of the facilities used by a public utility constitute a public utility?

HELD: EDSA LRT Corporation, Ltd. Is admittedly a foreign corporation duly incorporated and existing under the laws of Hong Kong. However, there is no dispute that once the EDSA LRT III is constructed, the private respondent, as lessor, will turn it over to DOTC as lessee, for the latter to operate the system and pay rentals for the said use. What private respondent owns are the rail tracks, rolling stocks, rail stations, terminals and the power plant, not a public utility. While a franchise is needed to operate these facilities to serve the public, they do not themselves constitute a public utility. What constitutes a public utility in not their ownership but their use to serve the public. The Constitution, in no uncertain terms, requires a franchise for the operation of a public utility. However, it does not require a franchise before one can own the facilities needed to operate a public utility so long as it does not operate them to serve the public. In law, there is a clear distinction between the operation of a public utility and the ownership of the facilities and the equipment used to serve the public.

41.

TATAD VS. GARCIA 243 SCRA 436 (1995)

FACTS: DOTC planned to construct a light railway transit line along Edsa. EDSA LRT Corporation, Ltd., a foreign corporation was awarded the contract to build, lease and transfer the said light railway. The said award was questioned by the petitioners on the basis that a foreign corporation cannot own the EDSA LRT III, a public utility as it violates the Constitution.

ISSUE: Whether or not an owner and lessor of the facilities used by a public utility constitute a public utility?

HELD: EDSA LRT Corporation, Ltd. Is admittedly a foreign corporation duly incorporated and existing under the laws of Hong Kong. However, there is no dispute that once the EDSA LRT III is constructed, the private respondent, as lessor, will turn it over to DOTC as lessee, for the latter to operate the system and pay rentals for the said use. What private respondent owns are the rail tracks, rolling stocks, rail stations, terminals and the power plant, not a public utility. While a franchise is needed to operate these facilities to serve the public, they do not themselves constitute a public utility. What constitutes a public utility in not their ownership but their use to serve the public? The Constitution, in no uncertain terms, requires a franchise for the operation of a public utility. However, it does not require a franchise before one can own the facilities needed to operate a public utility so long as it does not operate them to serve the public. In law, there is a clear distinction between the operation of a public utility and the ownership of the facilities and the equipment used to serve the public.

42.

TELECOM VS. COMELEC 289 SCRA 337 (1998)

FACTS: Petitioner Telecommunications and Broadcast Attorneys of the Philippines, Inc. (TELEBAP) is an organization of lawyers of radio and television broadcasting companies. It was declared to be without legal standing to sue in this case as, among other reasons, it was not able to show that it was to suffer from actual or threatened injury as a result of the subject law. Petitioner GMA Network, on the other hand, had the requisite standing to bring the constitutional challenge. Petitioner operates radio and television broadcast stations in the Philippines affected by the enforcement of Section 92, B.P. No. 881. Petitioners challenge the validity of Section 92, B.P. No. 881 which provides: Comelec Time- The Commission shall procure radio and television time to be known as the Comelec Time which shall be allocated equally and impartially among the candidates within the area of coverage of all radio and television stations. For this purpose, the franchise of all radio broadcasting and television stations are hereby amended so as to provide radio or television time, free of charge, during the period of campaign. Petitioner contends that while Section 90 of the same law requires COMELEC to procure print space in newspapers and magazines with payment, Section 92 provides that air time shall be procured by COMELEC free of charge. Thus it contends that Section 92 singles out radio and television stations to provide free air time. Petitioner claims that it suffered losses running to several million pesos in providing COMELEC Time in connection with the 1992 presidential election and 1995 senatorial election and that it stands to suffer even more should it be required to do so again this year. Petitioners claim that the primary source of revenue of the radio and television stations is the sale of air time to advertisers and to require these stations to provide free air time is to authorize unjust taking of private property. According to petitioners, in 1992 it lost P22,498,560.00 in providing free air time for one hour each day and, in this years elections, it stands to lost P58,980,850.00 in view of COMELECs requirement that it provide at least 30 minutes of prime time daily for such.

ISSUE: Whether of not Section 92 of B.P. No. 881 denies radio and television broadcast companies the equal protection of the laws.

HELD: Petitioners argument is without merit. All broadcasting, whether radio or by television stations, is licensed by the government. Airwave frequencies have to be allocated as there are more individuals who want to broadcast that there are frequencies to assign. Radio and television broadcasting companies, which are given franchises, do not own the airwaves and frequencies through which they transmit broadcast signals and images. They are merely given the temporary privilege to use them. Thus, such exercise of the privilege may reasonably be burdened with the performance by the grantee of some form of public service. In granting the privilege to operate broadcast stations and supervising radio and television stations, the state spends considerable public funds in licensing and supervising them.

43.

Associated Communications v NTC

FACTS: Act No. 3846, An Act Providing for the Regulation of Radio Stations and Radio Communications in the Philippines and for Other Purposes, was enacted. Pursuant to that, Congress enacted R.A. No. 4551, entitled An Act Granting Marcos J. Villaverde, Jr. and Winfred E. Villaverde a Franchise to Construct, Install, Maintain and Operate Public Radiotelephone and Radiotelegraph Coastal Stations, and Public Fixed and Public Based and Land Mobile Stations within the Philippines for the Reception and Transmission of Radiotelephone and Radiotelegraph for Domestic Communications and Provincial Telephone Systems in Certain Provinces. The franchise was transferred to petitioner Associated Communications & Wireless Services United Broadcasting Network, Inc. (ACWS for brevity) Petitioner ACWS then engaged in the installation and operation of several radio stations.P.D. No. 576-A, Regulating the Ownership and Operation of Radio and Television Stations and for other Purposes was issued: Sec. 6. All franchises, grants, licenses, permits, certificates or other forms of authority to operate radio or television broadcasting systems shall terminate on December 31, 1981. Thereafter, irrespective of any franchise, grant, license, permit, certificate or other forms of authority to operate granted by any office, agency or person, no radio or television station shall be authorized to operate without the authority of the Board of Communications and the Secretary of Public Works and Communications or their successors who have the right and authority to assign to qualified parties frequencies, channels or other means of identifying broadcasting system. A few years later, E.O. No. 546 integrated the Board of Communications and the Telecommunications Control Bureau under the Integrated Reorganization Plan into the NTC. Upon termination of petitioners franchise on December 31, 1981 pursuant to P.D. No. 576-A, it continued operating its radio stations under permits granted by the NTC.

As these presidential issuances relating to the radio and television broadcasting industry brought about confusion as to whether the NTC could issue permits to radio and television broadcast stations without legislative franchise. It is believed that the termination of all franchises granted for the operation of radio and television broadcasting systems effective December 31, 1981 and the vesting of the power to authorize the operation of any radio or television station upon the Board of Communications and the Secretary of Public Works and Communications and their successors under P.D. No. 576-A does not necessarily imply the abrogation of the requirement of obtaining a franchise under Act No. 3846. Under Act No. 3846, a radio broadcasting station must obtain a franchise from Congress. What was transferred to the Board of Communications and the Secretary of Commerce and Industry under P.D. No. 576-A was merely the regulatory powers vested solely in the Secretary of Commerce and Industry under Act No. 3846. E.O. No. 546 integrated the Board of Communications and the Telecommunications Bureau into a single entity known as the NTC and vested the new body with broad powers, among them, the power to issue Certificates of Public Convenience for the operation of communications utilities, including radio and televisions broadcasting systems and the power to grant permits for the use of radio frequencies. The NTC, the Committee on Legislative Franchises of Congress, and the KapisananngmgaBrodkastersaPilipinas of which petitioner is a member of good standing, entered into a Memorandum of Understanding (MOU) that requires a congressional franchise to operate radio and television stations. The parties hereto have agreed that NTC shall continue to issue and grant permits or authorizations to operate radio and television broadcast stations under EO No. 546. Prior to the December 31, 1994 deadline set by the MOU, petitioner filed with Congress an application for a franchise. Pending its approval, the NTC issued to petitioner a

temporary permit to operate a television station via Channel 25 of the UHF. It also granted petitioner a permit to purchase radio transmitters/transceivers for use in its television broadcasting. Shortly before the expiration of its temporary permit, petitioner applied for its renewal.

The franchise application of ACWS was not deliberated. NTCs Broadcast Service Department ordered petitioner to submit a new congressional franchise for the operation of its seven radio stations and that pending compliance, its application for temporary permits would be held in abeyance. Despite the absence of a congressional franchise, the NTC notified petitioner that its application for renewal of temporary permit was approved. After paying the fees, the NTC refused to release to petitioner its renewed permit. Instead, the NTC commencedagainst petitioner Administrative Case. Petitioner filed before the Court of Appeals a petition to compel the NTC to release its temporary permit. The appellate court denied The NTC rendered a decision on Administrative Case against petitioner. Hence, this petition

ISSUE:

Whether or not congressional franchise is a condition sine qua non in the operation of a radio and television broadcasting system.

HELD:

The power to issue franchises for radio and television systems is legislative in nature but is delegable. E.O. No. 546 is one law which authorizes an administrative agency, the NTC, to issue authorizations for the operations of radio and television broadcasting systems without need of a prior franchise issued by the Congress.

44.

Eastern Telecom v. Telecom Technologies

FACTS: President Fidel V. Ramos issued EO No. 109, the Local Exchange Carrier Service. It provides that all existing International Gateway Facility (IGF) operators are required to provide local exchange carrier services in un served and underserved areas, thereby promoting universal access to basic telecommunications service.NTC promulgated a Memorandum implementing the objectives of E.O. No. 109 which mandates existing IGF operators to file a petition for the issuance of Certificate of Public Convenience and Necessity (CPCN) to install, operate and maintain local exchange carrier services. To cap the government's efforts, RA No. 7925, otherwise known as the Public Telecommunications Policy Act of the Philippines, was enacted which mandates an international carrier to comply with its obligation to provide local exchange service in un served or underserved areas. NTC issued the Implementing Rules and Regulations for R.A. No. 7925.Taking advantage of the opportunities brought about by the passage of these laws, several IGF operators applied for CPCN to install, operate and maintain local exchange carrier services in certain areas. Communication Corporation (ICC)now known as Respondent, International Bayan Telecommunications

Corporation or Bayantel, applied for and was given by the NTC a Provisional Authority (PA). Meanwhile, petitioner Telecommunications Technologies Philippines, Inc. (TTPI), as an affiliate of petitioner Eastern Telecommunications Philippines, Inc. (ETPI), was granted by the NTC a PA.It appears, however, that before TTPI was able to fully accomplish its rollout obligation, ICC applied for and was given a PA by the NTC to install, operate and maintain a local exchange service in areas which were already covered by TTPI under its PA.

Aggrieved, petitioners filed a petition for review with the Court of Appeals arguing that the NTC committed grave abuse of discretion in granting a provisional authority to respondent ICC to operate in areas already assigned to TTPI.

ISSUE: Can a public utility franchise be exclusive?

HELD: No. Neither the Congress nor the NTC can grant an exclusive franchise certificate or any other form of authorization to operate a public utility. The Constitution is quite emphatic that the operation of the public utility shall not be exclusive. So Section 11, Article XII of the Constitution provides.Section 23 of EO 109 does not categorically state that the issuance of a PA is exclusive to any telecommunications company.

45.

GAMBOA vs. FINANCE SECRETARY TEVES G.R. No. 176579 June 28, 2011

FACTS: This is a petition to nullify the sale of shares of stock of Philippine Telecommunications Investment Corporation (PTIC) by the government of the Republic of the Philippines, acting through the Inter-Agency Privatization Council (IPC), to Metro Pacific Assets Holdings, Inc. (MPAH), an affiliate of First Pacific Company Limited (First Pacific), a Hong Kong-based investment management and holding company and a shareholder of the Philippine Long Distance Telephone Company (PLDT). The petitioner questioned the sale on the ground that it also involved an indirect sale of 12 million shares (or about 6.3 percent of the outstanding common shares) of PLDT owned by PTIC to First Pacific. With the this sale, First Pacifics common shareholdings in PLDT increased from 30.7 percent to 37 percent, thereby increasing the total common shareholdings of foreigners in PLDT to about 81.47%. This, according to the petitioner, violates Section 11, Article XII of the 1987 Philippine Constitution which limits foreign ownership of the capital of a public utility to not more than 40%. ISSUE: Does the term capital in Section 11, Article XII of the Constitution refer to the total common shares only, or to the total outstanding capital stock (combined total of common and non-voting preferred shares) of PLDT, a public utility? HELD: The term capital in Section 11, Article XII of the 1987 Constitution refers only to shares of stock entitled to vote in the election of directors, and thus in the present case only to common shares, and not to the total outstanding capital stock (common and non-voting preferred shares). Respondent Chairperson of the Securities and Exchange Commission is directed to apply this definition of the term capital in determining the extent of allowable foreign ownership in respondent Philippine Long Distance Telephone Company, and if there is a violation of Section 11, Article XII of the Constitution, to impose the appropriate sanctions under the law.

46.

PAGCOR VS. BIR 645 SCRA 338

FACTS: Petitioner Philippine Amusement and Gaming Corporation (PAGCOR), seeks to nullify Section 1 of Republic Act (R.A.) No. 9337 insofar as it amends Section 27 (c) of the National Internal Revenue Code of 1997, by excluding petitioner from exemption from corporate income tax for being repugnant to Sections 1 and 10 of Article III of the Constitution. Petitioner further seeks to prohibit the implementation of Bureau of Internal Revenue (BIR) Revenue Regulations No. 16-2005 for being contrary to law.

ISSUE: Is Republic Act 9337 constitutional insofar as it excluded PAGCOR from the enumeration of GOCCs exempt from the payment of corporate income tax?

HELD: YES. The original exemption of PAGCOR from corporate income tax was not made pursuant to a valid classification based on substantial distinctions so that the law may operate only on some and not on all. Instead, the same was merely granted due to the acquiescence of the House Committee on Ways and Means to the request of PAGCOR. The argument that the withdrawal of the exemption also violates the nonimpairment clause will not hold since any franchise is subject to amendment, alteration or repeal by Congress. However, the Court made it clear that PAGCOR remains exempt from payment of indirect taxes and as such its purchases remain not subject to VAT, reiterating the rule laid down in the Acesite case.

47.

NDC VS PVB 192 SCRA 575 (1990)

FACTS: P.D. 1717 was passed to rehabilitate the Agrix Group of Companies which neither a corporation owned nor controlled by government. As part of the rehabilitation process, the Agrix Group was dissolved by the decree and the decree created New Agrix, Inc. Likewise neither owned nor controlled by the government.

ISSUE: Does the decree creating the New Agrix, Inc valid? HELD: No. PD 1717 violates Article XIV, Section 4 of the 1973 Constitution now modelified as Art. XII, Sec. 16 it was passed to rehabilitate the Agrix Group of Companies which was not a GOCC.

48.

AGAN VS PIATCO 420 SCRA 257

FACTS: The DOTC and PIATCO entered into a concession agreement in 1997 to franchise and operate NAIA Airport terminal for 21years. In Nov. 1998 it was amended in the matters of pertaining to the definition of the obligations given to the concessionaire, development of facilities and proceeds, fees and charges, and the termination of contract. Since MIAA is charged with the maintenance and operations of NAIA terminals I and II, it has a contract with several service providers. The workers filed the petition for prohibition claiming that they would lose their job, and the service providers joined them, filed a motion for intervention. Likewise several employees of the MIAA filed a petition assailing the legality of arrangements. A group of congressmen filed similar petitions. Pres. Arroyo declared in her speech that she will not honor PIATCO contracts which the Exec. Branch's legal office concluded null and void. ISSUE:

Whether or Not the 1997 concession agreement is void, together with its amendments for being contrary to the constitution.

HELD: Public interest on the occasion of a national emergency is the primary consideration when the government decides to temporarily take over or direct the operation of a public utility or a business effected with public interest. The nature and extend of the emergency is the measure of the duration of the take over as well as the terms thereof. It is the states that prescribe the reasonable terms which will guide the implementation of the temporary take over. As dictated by the exigencies of time.

49.

DAVID VS MACAPAGAL-ARROYO GR NO 171396 MAY 2006

FACTS: These 7 consolidated petitions question the validity of PP 1017 (declaring a state of national emergency) and General Order No. 5 issued by President Gloria MacapagalArroyo. While the cases are pending, President Arroyo issued PP 1021, declaring that the state of national emergency has ceased to exist, thereby, in effect, lifting PP 1017. ISSUE:

Whether or not PP 1017 and G.O. No. 5 arrogated upon the President the power to enact laws and decrees If so, whether or not PP 1017 and G.O. No. 5 are unconstitutional

HELD: During existence of state of national emergency, PP 1017 purports to give the President, without authority/delegation from congress, to take over or direct the operation of any privately owned public utility/business affected with public interest. However, since emergency are reposed in Congress, sec 17 refers to Congress, not President. If the president wants to exercise such power, it must be delegated by congress. PP1017 therefore, does not authorize take over without authority from congress.

50.

DAVID VS. ARROYO GR No. 171396 May 2006

FACTS: In February 2006, due to the escape of some Magdalo members and the discovery of a plan (Oplan Hackle I) to assassinate GMA she declared PP 1017 and is to be implemented by GO 5. The said law was aimed to suppress lawlessness and the connivance of extremists to bring down the government. Pursuant to such PP, GMA cancelled all plans to celebrate EDSA I and at the same time revoked all permits issued for rallies and other public organization/meeting. Notwithstanding the cancellation of their rally permit, KMU head Randolf David proceeded to rally which led to his arrest. Later that day, the Daily Tribune, which Cacho-Olivares is the editor, was raided by the CIDG and they seized and confiscated anti-GMA articles and write ups. Later still, another known anti-GMA news agency (Malaya) was raided and seized. On the same day, Beltran of Anakpawis, was also arrested. His arrest was however grounded on a warrant of arrest issued way back in 1985 for his actions against Marcos. His supporters cannot visit him in jail because of the current imposition of PP 1017 and GO 5. In March, GMA issued PP 1021 w/c declared that the state of national emergency ceased to exist. David and some opposition Congressmen averred that PP1017 is unconstitutional for it has no factual basis and it cannot be validly declared by the president for such power is reposed in Congress. Also such declaration is actually a declaration of martial law. Olivares-Cacho also averred that the emergency contemplated in the Constitution is those of natural calamities and that such is an over breadth. Petitioners claim that PP 1017 is an over breadth because it encroaches upon protected and unprotected rights. The Sol-Gen argued that the issue has become moot and academic by reason of the lifting of PP 1017 by virtue of the declaration of PP 1021. The Sol-Gen averred that PP 1017 is within the presid ents calling out power, take care power and take over power.

ISSUE: Whether or not the president cannot validly order the taking over of private corporations or institutions such as the Daily Tribune without any authority from Congress.

HELD: The president cannot validly order the taking over of private corporations or institutions such as the Daily Tribune without any authority from Congress. On the other hand, the word emergency contemplated in the constitution is not limited to natural calamities but rather it also includes rebellion. The SC made a distinction; the president can declare the state of national emergency but her exercise of emergency powers does not come automatically after it for such exercise needs authority from Congress. The authority from Congress must be based on the following: 1 There must be a war or other emergency. (2) The delegation must be for a limited period only. (3) The delegation must be subject to such restrictions as the Congress may prescribe. (4) The emergency powers must be exercised to carry out a national policy declared by Congress.

51.

Republic v PLDT

FACTS:

Public petitioner commenced a suit against private respondent praying for the right of the Bureau of Telecommunications to demand interconnection between the Government Telephone System and that of PLDT, so that the Government Telephone System could make use of the lines and facilities of the PLDT. Private respondent contends that it cannot be compelled to enter into a contract where no agreement is had between them.

ISSUE: Under Section 18 may the state compel a public utility to render service in the public interest? HELD: Yes, in the exercise of the sovereign power of eminent domain, the Republic may require the telephone company to permit interconnection as the needs of the government service may require, subject to the payment of just compensation. The use of lines and services to allow inter-service connection between the both telephone systems, through expropriation can be a subject to an easement of right of way.

52.

REPUBLIC VS. PLDT 26 SCRA 620 (1968)

FACTS: PLDT and RCA Communications Inc (which is not a party to this case but has contractual relations with e parties) entered into an agreement where telephone messages, coming from the US and received by RCA's domestic station could automatically be transferred to the lines of PLDT and vice versa. The Bureau of Telecommunications set up its own Government Telephone Systemby renting the trunk lines of PLDT to enable government offices to call private parties. One of the many rules prohibits the use of the service for his private use. Republic of the Philippines entered into an agreement with RCA for a joint overseas telephone service where the Bureau would convey radio-telephone overseas calls received by the RCA's station to and from local residents.PLDT complained that the Bureau was violating the conditions for using the trunk lines not only for the use of government offices but even to serve private persons or the general public. PLDT gave a notice that if violations were not stopped, PLDT would sever the connections -which PLDT did. Republic sued PLDT commanding PLDT to execute a contract, through the Bureau, for the use of the facilities of defendant's telephone system throughout the Philippines under such terms and conditions as the court finds it reasonable. ISSUE: Whether or not Republic can command PLDT to execute the contract. HELD: No. The Bureau was created in pursuance of a state policy reorganizing the government offices to meet the exigencies attendant upon the establishment of a free Gov't of the Phil. When the Bureau subscribed to the trunk lines, defendant knew or should have known that their use by the subscriber was more or less public and all embracing in nature. The acceptance by the defendant of the payment of rentals, despite its knowledge that the plaintiff had extended the use of the trunk lines to commercial purposes, implies assent by the defendant to such extended use. Since this relationship has been maintained for a long time and the public has patronized both telephone systems, and their interconnection is to the public convenience, it is too late for the defendant to claim misuse of its facilities, and it is not now at liberty to unilaterally sever the physical connection of the trunk lines. To uphold PLDT's contention is to subordinate the needs of the general public.

53.

ENERGY REGULATORY BOARD, vs. COURT OF APPEALS and PETROLEUM DISTRIBUTORS AND SERVICES CORPORATION, G.R. No. 113079 April 20, 2001 PILIPINAS SHELL PETROLEUM CORPORATION, vs. COURTOF APPEALS and PETROLEUM DISTRIBUTORS AND SERVICESCORPORATION, G.R. No. 114923 April 20, 2001

FACTS: The propriety of building a state-of-the-art gasoline service stationalong Benigno Aquino, Jr. Avenue in Paranaque, Metro Manila isthe bone of contention in these consolidated petitions for certiorari under Rule 45 of the Rules of Court. Petitioners assert that the construction of such a modern edifice is a necessity dictated bythe "emerging economic landscapes." Respondents say otherwise. Petitioner Pilipinas Shell Petroleum Corporation (Shell) is engaged in the business of importing crude oil, refining the same and selling various petroleum products through a network of service stations throughout the country. Private respondent Petroleum Distributors and Service Corporation(PDSC) owns and operates a Caltex service station at the corner of the MIA and Domestic Roads in Pasay City. On June 30,1983, Shell filed with the quondam Bureau of Energy Utilization (BEU) an application for authority to relocate its Shell Service Station at Tambo, Paranaque, Metro Manila, to Imelda Marcos Avenue of the same municipality. The application, which was docketed as BEU Case No. 83-09-1319, was initially rejected by the BEU because Shell's old site had been closed for five (5)years such that the relocation of the same to a new site would amount to a new construction of a gasoline outlet, which construction was then the subject of a moratorium. Subsequently, however, BEU relaxed its position and gave due course to the application.

PDSC filed an opposition to the application on the grounds that:1.] there are adequate service stations attending to the motorists'requirements in the trading area covered by the application; 2.]ruinous competition will result from the establishment of theproposed new service station; and 3.] there is a decline not anincrease in the volume of sales in the area. Two other companies,namely Petrophil and Caltex, also opposed the application on theground that Shell failed to comply with the jurisdictionalrequirements. On June 3, 1986, the BEU rendered a decision denying Shells application. Meanwhile, on May 8, 1987, Executive Order No. 172was issued creating the Energy Regulatory Board (ERB) andtransferring to it the regulatory and adjudicatory functions of theBEU. On May 9, 1988, the OEA rendered a decision denying the appeal of Shell and affirming the BEU decision. Shell moved forreconsideration and prayed for a new hearing or the remand of the case for further proceedings. In a supplement to said motion, Shell submitted a new feasibility study to justify its application. The OEA issued an order on July 11, 1988, remanding the case to the ERB for further evaluation and consideration, noting thereinthat the "updated survey conducted by Shell" cited new developments such as the accessibility of Imelda Marcos Avenue, now Benigno Aquino, Jr. Avenue, to Paraaque residents alongSucat Road and the population growth in the trading area. After the records of BEU Case No. 83-09-1319 was remanded tothe ERB, Shell filed on March 3, 1989 an amended application, intended for the same purpose as its original application. This amended application was likewise opposed by PDSC. On September 17, 1991, the ERB rendered a Decision allowing Shell to establish the service station in Benigno Aquino, Jr.Avenue. PDSC filed a motion for reconsideration of the foregoing Decision. The motion was, however, denied. PDSC elevated its cause on to the Court of Appeals.CA reversed the ERB judgment. A motion for reconsideration was denied by the Court of Appeals in a Resolution dated 6 April 1994.Dissatisfied, both Shell andERB elevated the matter to this Court by way of these petitions, which were ordered consolidated by the Court in a Resolution dated July 25,1994.

ISSUE WON THE ESTABLISHMENT OF THE SERVICE STATION WILL NOT LEAD TO RUINOUS COMPETITION. HELD: A distinct worldwide trend towards economic deregulation has been evident in the past decade. Both developed and developing countries have seriously considered and extensively adopted various measures for this purpose. The country has been no exception. Indeed, the buzzwords of the third millenium are "deregulation", "globalization" and "liberalization."19 It need not be overemphasized that this trend is reflected in our policy considerations, statutes and jurisprudence. Thus, in Garcia v. Corona,20 the Court said: R.A. 8479, the present deregulation law, was enacted to implement Article XII, Section 19 of the Constitution which provides:The State shall regulate or prohibit monopolies when the public interest so requires. No combinations in restraint of trade or unfair competition shall be allowed. This is so because the Government believes that deregulation will eventually prevent monopoly. The simplest form of monopoly exists when there is only one seller or producer of a product or service for which there are no substitutes. In its more complex form, monopoly is defined as the joint acquisition or maintenance by members of a conspiracy, formed for that purpose, of the power to control and dominate trade and commerce in a commodity to such an extent that they are able, as a group, to exclude actual or potential competitors from the field, accompanied with the intention and purpose to exercise such power.21 It bears reiterating at the outset that deregulation of the oil industry is policy determination of the highest order. It is unquestionably a priority program of Government. The Department of Energy Act of 199222 expressly mandates that the development and updating of the existing Philippine energy program "shall include a policy direction towards deregulation of the power and energy industry.

54.

ENRIQUE GARCIA VS EXECUTIVE SECRETARY

FACTS: Cory issued Executive Order 438 which imposed, in addition to any other duties, taxes and charges imposed by law on all articles imported into the Philippines, an additional duty of 5% ad valorem. This additional duty was imposed across the board on all imported articles, including crude oil and other oil products imported into the Philippines. In 1991, EO 443 increased the additional duty to 9%. In the same year, EO 475 was passed reinstating the previous 5% duty except that crude oil and other oil products continued to be taxed at 9%. Garcia, a representative from Bataan, avers that EO 475 and 478 are unconstitutional for they violate Sec 24 of Art 6 of the Constitution which provides: All appropriation, revenue or tariff bills, bills authorizing increase of the public debt, bills of local application, and private bills shall originate exclusively in the House of Representatives, but the Senate may propose or concur with amendments. He contends that since the Constitution vests the authority to enact revenue bills in Congress, the President may not assume such power of issuing Executive Orders Nos. 475 and 478 which are in the nature of revenue-generating measures. ISSUE: Whether or not EO 475 and 478 are constitutional. HELD: Under Section 24, Article VI of the Constitution, the enactment of appropriation, revenue and tariff bills, like all other bills is, of course, within the province of the Legislative rather than the Executive Department. It does not follow, however, that therefore Executive Orders Nos. 475 and 478, assuming they may be characterized as revenue measures, are prohibited to the President, that they must be enacted instead by the Congress of the Philippines. Section 28(2) of Article VI of the Constitution provides as follows: (2) The Congress may, by law, authorize the President to fix within specified limits, and subject to such limitations and restrictions as it may impose, tariff rates, import and export quotas, tonnage and wharf age dues, and other duties or imposts within the framework of the national development program of the Government. There is thus explicit constitutional permission to Congress to authorize the President subject to such limitations and restrictions as [Congress] may impose to fix w ithin specific limits tariff rates . . . and other duties or imposts . . . .

55.

TATAD V. SECRETARY OF ENERGY 281 SCRA 330

FACTS: The petitions assail the constitutionality of various provisions of RA 8180 entitiled the Downstream Oil Industry Deregulation Act of 1996. Under the deregulated environment, any person or entity may import or purchase any quantity of crude oil and petroleum products from a foreign or domestic source, lease or own and operate refineries and other downstream oil facilities and market such crude oil or use the same for his own requirement, subject only to monitoring by the Department of Energy.

ISSUES:

(1) Whether or not the petitions raise a justiciable controversy (2) Whether or not the petitioners have the standing to assail the validity of the law (3) Whether or not Sec. 5(b) of RA 8180 violates the one title one subject requirement of the Constitution (4) Whether or not Sec. 15 of RA 8180 violates the constitutional prohibition on undue delegation of power (5) Whether or not RA 8180 violates the constitutional prohibition against monopolies, combinations in restraint of trade and unfair competition

Held: As to the first issue, judicial power includes not only the duty of the courts to settle actual controversies involving rights which are legally demandable and enforceable, but also the duty to determine whether or not there has been grave abuse of discretion amounting to lack or excess of jurisdiction on the part of any branch or instrumentality of the government. The courts, as guardians of the Constitution, have the inherent authority to determine whether a statute enacted by the legislature transcends the limit imposed by the fundamental law. Where a statute violates the Constitution, it is not only the right but the duty of the judiciary to declare such act as unconstitutional and void.

The effort of respondents to question the legal standing of petitioners also failed. The Court has brightlined its liberal stance on a petitioners locus standi where the petitioner is able to craft an issue of transcendental significance to the people. In the case, petitioners pose issues which are significant to the people and which deserve the Courts forthright resolution. It is also contended that Sec. 5(b) of RA 8180 on tariff differential violates the provision of the Constitution requiring every law to have only one subject which should be expressed in its title. The Court did not concur with this contention. The title need not mirror, fully index or catalogue all contents and minute details of a law. A law having a single general subject indicated in the title may contain any number of provisions, no matter how diverse they may be, so long as they are not inconsistent with or foreign to the general subject, and may be considered in furtherance of such subject by providing for the method and means of carrying out the general subject. The Court held that Sec. 5 providing for tariff differential is germane to the subject of RA 8180 which is the deregulation of the downstream oil industry. Petitioners also assail Sec. 15 of RA 8180 which fixes the time frame for the full deregulation of the downstream oil industry for being violative of the constitutional prohibition on undue delegation of power. There are two accepted tests to determine whether or not there is a valid delegation of legislative power: the completeness test and the sufficient standard test. Under the first test, the law must be complete in all its terms and conditions when it leaves the legislative such that when it reaches the delegate the only thing he will have to do is to enforce it. Under the sufficient standard test, there must be adequate guidelines or limitations in the law to map out the boundaries of the delegates authority and prevent the delegation from running riot. Section 15 can hurdle both the completeness test and the sufficient standard test. Congress expressly provided in RA 8180 that full deregulation will start at the end of March 1997, regardless of the occurrence of any event. Full deregulation at the end of March 1997 is mandatory and the Executive has no discretion to postpone it for any purported reason. Thus, the law is complete on the question of the final date of full deregulation. The discretion given to the President is to advance the date of full deregulation before the end of March 1997. Section 15 lays down the standard to guide the judgment of the President. He is to time it as far as practicable when the prices of crude oil and petroleum products in the world market are declining and when the exchange rate of the peso in relation to the US dollar is stable.

Petitioners also argued that some provisions of RA 8180 violate Sec. 19, Art. XII of the Constitution. Section 19, Art. XII of the Constitution espouses competition. The desirability of competition is the reason for the prohibition against restraint of trade, the reason for the interdiction of unfair competition, and the reason for regulation of unmitigated monopolies. Competition is thus the underlying principle of Sec. 19, Art. XII of the Constitution which cannot be violated by RA 8180. Petron, Shell and Caltex stand as the only major league players in the oil market. As the dominant players, they boast of existing refineries of various capacities. The tariff differential of 4% on imported crude oil and refined petroleum products therefore works to their immense benefit. It erects a high barrier to the entry of new players. New players that intend to equalize the market power of Petron, Shell and Caltex by building refineries of their own will have to spend billions of pesos. Those who will not build refineries but compete with them will suffer the huge disadvantage of increasing their product cost by 4%. They will be competing on an uneven field. The provision on inventory widens the balance of advantage of Petron, Shell and Caltex against prospective new players. Petron, Shell and Caltex can easily comply with the inventory requirement of RA 8180 in view of their existing storage facilities. Prospective competitors again will find compliance with this requirement difficult as it will entail a prohibitive cost.

The most important question is whether the offending provisions can be individually struck down without invalidating the entire RA 8180. The general rule is that where part of a statute is void as repugnant to the Constitution, while another part is valid, the valid portion, if separable from the invalid, may stand and be enforced. The exception to the general rule is that when the parts of a statute are so mutually dependent and connected, as conditions, considerations, inducements or compensations for each other, as to warrant a belief that the legislature intended them as a whole, the nullity of one part will vitiate the rest. RA 8180 contains a separability clause. The separability clause notwithstanding, the Court held that the offending provisions of RA 8180 so permeate its essence that the entire law has to be struck down. The provisions on tariff differential, inventory and predatory pricing are among the principal props of RA 8180. Congress could not have regulated the downstream oil industry without these provisions. Unfortunately, contrary to their intent, these provisions on tariff differential, inventory and predatory pricing inhibit fair competition, encourage monopolistic power and interfere with the free interaction of market forces.

56.

EASTERN ASSURANCE VS. LTFRB G. R. No. 149717 (Oct. 7, 2003)

FACTS:

The Board issued Memorandum Circular No. 99-011 fixing the insurance coverage of PUVs on the basis of the number of persons that may be killed or injured instead of the entire vehicle alone. The coverage is denominated as Passenger Accident Insurance Coverage (PAIC), which fixes the coverage of P50,000.00 per passenger. In order to make sure that future claims of PUV operators and passenger accident victims are paid within the required time, and in order to minimize, if not eliminate, fake certificates of cover and graft and corruption, as well as to ensure the payment of the proper taxes much needed by the government, as well as to create a computerized data bank without any cost to the government which is necessary for transport planning, the Board will only accept, as proof of compliance of this program, insurance policies/certificates of cover duly approved by the Insurance Commission specifically for this project, and issued by any of the two groups as authorized by the Board. Accordingly, as there is already one group duly authorized by the Board to participate in this program in the person of the Passenger Accident Managers, Inc. (PAMI for brevity), THERE IS A NEED TO FORM ANOTHER GROUP IN ORDER TO FULLY IMPLEMENT THE PROGRAM. All other insurance companies who wish to continue participating in the program, therefore, are hereby required to either join PAMI or form a second group. Petitioner contends that Memorandum Circular No. 2001-001 and the subsequent implementing Circulars violate the constitutional proscription against monopoly as well as unfair competition and combination in restraint of trade. Petitioner further argues that these were issued with grave abuse of discretion and without jurisdiction on the part of the LTFRB.

ISSUE: Whether or not the operation of monopolies is totally banned by the constitution.

HELD: The operation of monopolies is not totally banned by the Constitution. However, the State shall regulate them when public interest so requires. In the present case, the two consortia of insurance companies that have been authorized to issue passenger insurance policies are adequately regulated by the Land Transportation Franchising and Regulatory Board (LTFRB) to protect the riding public. While individual insurance companies may somehow be adversely affected by this scheme, the paramount public interest involved must be upheld. In any event, all legitimate insurance companies are allowed to become members of the consortia. Thus, there is no restraint of trade or unfair competition involved.

57.

AVON VS. LUNA G.R. NO. 153674 Dec 20, 2006

FACTS: In 1978 Avon acquired Beautifont Inc, a Philippine company, where Luna was an employee. In 1985, Luna and Avon signed the so-called Supervisor's Agreement. This had the following terms: (i) that the Agreement does not make the Supervisor an employee or agent of the Company (ii) that the Supervisor is an independent retailer/dealer and has sole discretion to determine where and how Avon's products will be sold, except that the Supervisor cannot sell such products to stores, supermarkets or to any person who sells things at a fixed place of business, (iii) that the Supervisor shall sell or offer to sell, display or promote only and exclusively products sold by the Company; (iv) that either party may terminate the agreement at will, or without cause, at any time upon notice to the other. In 1988, Luna became group franchise director of Sandre Philippines, Inc, which was engaged in direct sales of vitamins and other supplements. She began to sell and promote Sandre products to Avon employees and friends. On September 23 1988, Luna asked a law firm for an opinion regarding the legal consequences of the Supervisory Agreement she had entered into with Avon. The law firm said it was contrary to law and public policy. Luna then wrote to her colleagues attaching mimeographed copies of the legal opinion, and urged them to engage in other business without fear. On October 11 1988, Avon gave Luna notice of the termination of the Supervisory Agreement on the grounds that she had violated the exclusivity clause. ISSUE: Whether the Court of Appeals erred in ruling that the Supervisors Agreement was invalid for being contrary to public policy HELD: The Supreme Court observed that both Avon and Sandre employ direct selling to sell their products: "It was not by chance that Sandre Philippines made respondent Luna one of its Group Franchise Directors," the Court said in its ruling. "It doesn't take a genius to realize that by making her an important part of its distribution arm, Sandre Philippines, a newly formed direct selling business, would be saving time, effort and money as it will no longer have to train and motivate supervisors and dealers. Respondent Luna who learned the tricks of the trade from petitioner Avon, will do it for them. This is tantamount to unjust enrichment.

Worse, the goodwill established by Avon among its loyal customers will be taken advantage of by Sandre Philippines. It is not so hard to imagine the scenario wherein the sale of Sandre products by Avon dealers will engender a belief in the minds of loyal Avon customers that the products they are buying have been manufactured by Avon. In other words, they will be misled into thinking that the Sandre products are in fact Avon products. From the foregoing, it cannot be said that the purpose of the subject exclusivity clause is to foreclose the competition, that is the entrance of Sandre products into the market. Therefore, it cannot be considered void for being against public policy."

You might also like